Basic Mathematics 1
Basic Mathematics 1
Mamadou Ndao
M AMADOU N DAO
2
Introduction
This book is a course given to L0 students at UFAZ during the first semester of
the academic years 2019-2020, 2020-2021 and 2021-2022. The Frecnh-Azerbaijani
University called this year the foundation year because it allows to fill the gap that
between the French High School program and Azerbaijan High School’s program.
Its main objective is to give general backgrounds in mathematics to students who
integrate UFAZ, in order to allow them to be able to follow first year’s courses of
the University of Strasbourg. In other words this year corresponds to the ”Termi-
nale” of french program. This book is concerned by the first Semester. It consists
of integration courses and deals with complex numbers, generalities on sequences,
arithmetic and geometric sequences proof by induction, geometry in the plane and
geometry in space. In this book, we follow the following methodology: each chapter
is composed of three parts. In the main part of each chapter we recall the general
results of the topic and give examples that help to understand this subject. To bet-
ter understand the lecture we propose a list of exercises with their corrections. For
students who want to go further we add a list of exercises without any correction
in order to challenger them. We emphasize that most of the problems and exercises
come from the web site APMEP. They were given during the Baccalaureat in the
previous years. We took also some exercises from the list of homework given by
Fuad Aliyev (UFAZ) and Loic Célier (UFAZ).
ACKNOWLEDGE
My most sincere thanks go to my dear colleagues Ulviyya Abdulkarimova, Javanshir Azizov, Fuad
Aliyev and Sabina Samadova for their help and support
To A Group 2021
To my dear friend Allahverdi Gasimov
Elementary Mathematics I
Mamadou Ndao
II
CONTENTS
2 Introduction To Sequences 35
2.1 Generalities on Sequences . . . . . . . . . . . . . . . . . . . . . . . . 36
2.1.1 Computation of the Terms of a Sequence . . . . . . . . . . . 37
2.1.2 Bounded Sequences . . . . . . . . . . . . . . . . . . . . . . . 39
2.1.3 Monotonic Sequences . . . . . . . . . . . . . . . . . . . . . . 39
2.2 Proof by Induction . . . . . . . . . . . . . . . . . . . . . . . . . . . . 41
2.2.1 First Form of Induction . . . . . . . . . . . . . . . . . . . . . 41
2.2.2 Second Form of Induction . . . . . . . . . . . . . . . . . . . . 43
2.3 Study of Limits of Sequences . . . . . . . . . . . . . . . . . . . . . . 44
2.3.1 Definitions of Limits of Sequences . . . . . . . . . . . . . . . 44
2.3.2 General Theorems on Convergence . . . . . . . . . . . . . . . 46
2.3.3 Elementary Operations on Limits . . . . . . . . . . . . . . . . 53
2.4 Exercise Session . . . . . . . . . . . . . . . . . . . . . . . . . . . . . . 59
III
IV CONTENTS
2.5 Problems . . . . . . . . . . . . . . . . . . . . . . . . . . . . . . . . . 66
B Exams 191
VI CONTENTS
CHAPTER 1
INTRODUCTION TO COMPLEX
NUMBERS
Contents
1.1 Algebraic Form of a Complex Number . . . . . . . . . 2
1.1.1 Sum and Multiplication of Complex Numbers . . . . . . 3
1.1.2 Conjugate of a Complex Number . . . . . . . . . . . . . 4
1.1.3 Division of Complex Numbers . . . . . . . . . . . . . . . 6
1.1.4 Modulus of a complex number . . . . . . . . . . . . . . 7
1.2 Polar Form of a Complex Number . . . . . . . . . . . . 9
1.3 Exponential Form of a Complex Number . . . . . . . . 14
1.4 Second Order Polynomial Equations . . . . . . . . . . 15
1.4.1 Equation With Real Coefficients . . . . . . . . . . . . . 15
1.4.2 Equations With Complex Coefficients . . . . . . . . . . 17
1.5 Nth Roots of a Complex Numbers . . . . . . . . . . . . 20
1.5.1 The nth Roots of the Unit . . . . . . . . . . . . . . . . . 20
1.5.2 The nth Roots of a Complex Number . . . . . . . . . . 21
1.6 General Properties of Complex Numbers . . . . . . . . 22
1.7 Exercise Session . . . . . . . . . . . . . . . . . . . . . . . 23
1.8 Problems . . . . . . . . . . . . . . . . . . . . . . . . . . . 31
x2 + 1 = 0, (1.0.1)
we observe that there is no x ∈ R which verifies (1.0.1). The main goal of this
chapter is to introduce a set in which we can be able to define solutions to (1.0.1).
1
2 CHAPTER 1. INTRODUCTION TO COMPLEX NUMBERS
x2 − i2 = x − i x + i = 0.
(1.0.2)
In this case one finds that the solutions of (1.0.1) are x = i or x = −i.
The new field in which (1.0.1) admits solutions is called the set of complex
numbers and we denote it C.
To study complex numbers in details we proceed as follows. In section 1 we define
the algebraic form of a complex number. In section 2 and 3 we study respectively
the polar form and the exponential form of complex numbers. In section 4 we
investigate second order polynomial equations. The section 5 is devoted to some
general results on complex numbers. We end this chapter by a list of problems.
Definition 1.1.2. We say that , z is a complex number if there are two reals
numbers a and b, such that,
z = a + i b. (1.1.1)
The real numbers a and b are respectively called the real part and the imaginary
part of z. We denote a := Re(z) and b := Im(z).
Remark 1.1.3. In the formula (1.1.1),
1. when b = 0, then, z = a is a real number
2. when a = 0, z = i b is called a purely imaginary number.
Example 1.1.4. We consider the complex numbers z1 = 3 and z2 = 5 i.
We can see that z1 is real and z2 is purely imaginary number.
Definition 1.1.5. The formula (1.1.1) is called the algebraic form of the complex
number z.
Example 1.1.6. Let z = 4 − 3i. Then, we have Re(z) = 4 and Im(z) = −3.
Example 1.1.7. We consider the complex number z = −3 + i. In this case, we
have Re(z) = −3 and Im(z) = 1.
Exercise 1.1.8. Determine the real and the imaginary part of the following complex
numbers
If we take the horizontal line passing through 0 as the real axis and the vertical
line that passes through 0 as the imaginary axis, we can represent the complex
number z = a + ib in this way
Im
b •a + i b
a Re
Proof. if z1 and z2 are two complex numbers such that z1 = z2 , then, one has
z1 − z2 = Re(z1 ) − Re(z2 ) + i Im(z1 ) − Im(z2 ) = 0.
Example 1.1.11. The complex numbers z1 = 52 + 32 i and z2 = 2.5 + 1.5i are equal.
But the complex numbers z3 = 1 − i and z4 = 1 + i are different. Because
Im(z3 ) 6= Im(z4 ).
At this stage we cannot deal with division. Because to divide two complex
numbers, we need to define the conjugate of the complex number.
Im
×
z = a + ib
Re
z = a − ib
×
n
Theorem 1.1.21. If z is a complex number, then, z n = z .
2. z1 · z 2 − z 1 · z2 = 2 i Im(z1 · z 2 ).
6 CHAPTER 1. INTRODUCTION TO COMPLEX NUMBERS
Proof. We consider two complex numbers z and z 0 , such that, z 0 6= 0. Then, com-
bining proposition 1.1.20 and lemma 1.1.23, we have
z
1 1 z
0
= z · = z · 0 = 0·
z z z z
Im M (z = a + i b)
b× •
|z|
O a Re
z · z = |z|2 .
From this Lemma we deduce that two things: first we see that z · z ∈ R+ .
Secondly we have |z| = |z|.
2. |z n | = |z|n
1 1
3. =
z |z|
Proposition 1.1.36. If z and z 0 are two complex numbers such that z 0 6= 0, then
z |z|
= 0 ·
z0 |z |
Proof. We take two complex numbers z and z 0 such that z 0 6= 0. One sees that
z 1 |z|
0
= |z| · 0 = 0 ·
z |z | |z |
Lemma 1.1.37. For any natural numbers n, m and any complex numbers z and
z 0 such that z 0 6= 0, we have
zn |z|n
= ·
z0m |z 0 |m
Considering the square root of the inequality above, we obtain the point 3.
Im M (z = a + i b)
•
θ = arg(z)
O Re
θ := arg(z).
Remark 1.2.2. We draw the readers attention that an argument of the complex
number z is determined up on a multiple of 2π. This means that if θ is an argument
of the complex number z, then
θ + 2kπ with k ∈ Z,
√ √ √
Example 1.2.3. Let z = −1 + 3. We deduce that |z| = 1+3 = 4 = 2. If we
denote θ := arg(z), one obtains
√
1 3
cos(θ) = − and sin(θ) = ·
2 2
This implies that
π
arg(z) = π − ·
3
1.2. POLAR FORM OF A COMPLEX NUMBER 11
√
Example 1.2.4. We define z = −1 − i and θ := arg(z). Then, we obtain |z| = 2
and √ √
− 2 − 2
cos(θ) = and sin(θ) = ·
2 2
Therefore we have
π
arg(z) = π + ·
4
Using the argument of the complex number z and its modulus we can express
the complex number z in terms of cos and sin. This expression wil be called the
polar or Trigonometric form.
Definition 1.2.5. The polar or trigonometric form of z which has argument θ
is
z = |z| cos(θ) + i sin(θ) . (1.2.2)
Example 1.2.6. Define the polar or trigonometric forms of the complex numbers
√ √
z1 = 1 + i, z2 = 1 + i 3, z3 = −1, z4 = 3 + i.
√ √
Solution. 1. We consider z1 = 1+i, then |z1 | = 1 + 1 = 2. Defing θ := arg(z1 )
one obtains
√ √
1 2 1 2
cos(θ) = √ = and sin(θ) = √ = ·
2 2 2 2
This leads to
π
θ= + 2 k π,
4
with some k ∈ Z. Hence the polar form of z1 ,
√ π π
z1 = 2 cos + i sin .
4 4
√ √
2. For z2 = 1 + i 3, one has |z2 | = 1 + 3 = 2. Taking θ := arg(z2 ), we have
√
1 3
cos(θ) = and sin(θ) = ·
2 2
Therefore we obtain,
π
θ = + 2kπ, where k ∈ Z.
3
We conclude that the trigonometric form of z2 is
π π
z2 = 2 cos + i sin .
3 3
3. We consider the complex number z3 = −1. We can see |z3 | = 1. Now we fix
θ := arg(z3 ). This means that
cos(θ) = −1 and sin(θ) = 0.
In this case
θ = π + 2kπ, with k ∈ Z.
The polar form of z3 is
z3 = cos(π).
12 CHAPTER 1. INTRODUCTION TO COMPLEX NUMBERS
√
4. Let z4 = 3 + i. We have |z4 | = 2. Setting θ := arg(z4 ) the following properties
hold:
√3 1
cos θ = and sin θ = ·
2 2
Therefore one has
π
θ = + 2kπ, with k ∈ Z.
6
Hence the trigonometric form is
π π
z4 = 2 cos + i sin .
6 6
Exercise 1.2.7. Determine the polar form of the following complex numbers
√ √ √ √
− 1 − i, 1 − i 3, 3 − i, − 3 + i, −2 + 2 i, (1 + i)( 3 − i), −3i, 2.
Proposition 1.2.9. If z and z 0 are two complex numbers such that z 0 6= 0, then
we have
1. arg z10 = arg z 0 = −arg(z 0 ),
z0
1
arg = arg = arg(z 0 ) = −arg(z 0 ).
z0 |z 0 |2
1.2. POLAR FORM OF A COMPLEX NUMBER 13
To prove 2, we recall that z = |z| cos(θ1 ) + i sin(θ1 ) . This involves
z.z 0 = |z| |z 0 | cos(θ1 ) cos(θ2 ) + i sin(θ1 ) cos(θ2 ) + i cos(θ1 ) sin(θ2 ) − sin(θ1 ) sin(θ2 )
h i
= |z||z 0 | cos(θ1 ) cos(θ2 ) − sin(θ1 ) sin(θ2 ) + i sin(θ1 ) cos(θ2 ) + cos(θ1 ) sin(θ2 )
h i
= |z||z 0 | cos(θ1 + θ2 ) + i sin(θ1 + θ2 ) .
Example 1.2.10. Define the polar form of the following complex numbers:
√ 1+i
(1 + i)( 3 − i) and √ ·
3−i
√ √ π
Solution. We define z1 = 1+i and z2 = 3−i. In this case, |z1 | = 2, arg(z1 ) = 4,
|z2 | = 2 and arg(z2 ) = − π6 . Using the proposition above we conclude that
√
z1 · z2 = 2 2 cos π/12 + i sin π/12
√
z1 2 5π 5π
= cos + i sin .
z2 2 12 12
Exercise 1.2.11. For each of the following complex numbers define its polar form
√ √
√ − 3−i √ 1+i 1 2i 1−i 3
(1 − i)(2 + 2 3 i), , − 3 − i, , , √ , ·
−1 − i 5 − 5i 3i 3+i 2
In General one can prove the following lemma wich is a generalization of the
second point in the proposition above.
Lemma 1.2.12. For every complex numbers z1 , z2 , · · · , zn , we have
n
X
arg(z1 · z2 · · · · · zn ) = arg(z1 ) + · · · + arg(zn ) = arg (zk ) .
k=1
Using Euler’s formula we observe that for every real number θ we have
exp − i θ = cos(−θ) + i sin(−θ) = cos(θ) − i sin(θ) = exp i θ .
Lemma 1.3.2. Let z be a complex number with argument θ. Then, the exponen-
tial form of the conjugate of z is
z = |z| exp − i θ .
The most important theorem of this section is De Moivre’s theorem which runs in
this way:
Theorem 1.3.4 (De Moivre’s Theorem). For any complex number that has ar-
gument θ and any positive integer n we have
z n = |z|n cos(n θ) + i sin(n θ) (1.3.3)
Example 1.3.5. Determine the polar form of the following complex numbers
5 √ 7
1. 1 + i 2. 1 − i 3 .
√
Solution. We define z1 = 1 + i. In this case one obtains |z1 | = 2 and arg(z) = π4 ·
This implies that
√
5π 5π
z15 = 4 2 cos + i sin .
4 4
√ √
We take z2 = 1 − i 3. This means that |z2 | = 1 + 3 = 2 and arg(z2 ) = − π3 ·
Hence one has
7 7π 7π
z2 = 2 cos − + i sin − .
3 3
Exercise 1.3.6. Using De Moivre’s theorem determine the polar forms of the fol-
lowing complex numbers
√ 5 √ 11 √ 8 3 √ 6
− 3+i , −1−i 3 , − 3 − 3i 3 , 1−i , − 3−i .
az 2 + bz + c = 0 (1.4.1)
Theorem 1.4.1. If a, b and c are three real numbers such that a 6= 0 and we
define
∆ = b2 − 4a c,
then,
when ∆ > 0, equation (1.4.1) has two real solutions
√ √
−b − ∆ −b + ∆
z1 = or z2 = · (1.4.2)
2a 2a
when ∆ = 0, equation (1.4.1) has only one real solution, which is defined by
−b
z0 = · (1.4.3)
2a
∆ = b2 − 4ac = 16 − 12 = 4 > 0.
−(−2) 2
z0 = = = 1.
2 2
2. We consider the equation z 2 +6 z +9 = 0. Then, ∆ = 62 −4×9 = 36−36 = 0.
Therefore we have one real solution
−6
z0 = = −3.
2
1.4. SECOND ORDER POLYNOMIAL EQUATIONS 17
z 2 + 3 z + 2 = 0, z 2 + 6 z + 8 = 0, z 2 + 7 z + 9 = 0, z 2 + 4 z + 2 = 0.
z 2 + 2 z + 1 = 0, z 2 − 4 z + 4 = 0, z 2 − 6 z + 9 = 0, z 2 − 8 z + 16 = 0.
z 2 + z + 5 = 0, z 2 + 2 z + 4 = 0, z 2 + 3 z + 5 = 0, z 2 + 5 z + 7 = 0.
Remark 1.4.8. When ∆ < 0, the solutions z1 and z2 are two conjugate complex
numbers.
In the following subsection we will see how to study second order polynomial
equations with complex coefficients.
Proof. We fix the complex a + ib for two real numbers a and b. Let z = x + i y be a
complex number such that,
z 2 = a + i b. (1.4.5)
To determine the solutions of this equation we proceed in four steps
18 CHAPTER 1. INTRODUCTION TO COMPLEX NUMBERS
x2 − y 2 + 2i xy = a + i b.
2xy = b.
√
Step 2: We emphasize that |z|2 = |x + i y|2 = x2 + y 2 = a2 + b2 . This holds the
following identities
p
x2 − y 2 = a, x2 + y 2 = a2 + b2 and 2xy = b.
Step 4: Now we use the sign of b = 2 xy to determine the exact values of x and y.
Example 1.4.10. Find the square roots of the following complex numbers
1. 8 + 6 i, 2. 3 − 4 i.
+ 6 i. Then we have z 2 = 8 + 6 i.
Solution. 1.Let z = x + i y be a square root of 8 √
2 2 2 2
We obtain x − y = 8 , 2x y = 6, and x + y = 82 + 62 = 10. We have
√ √
2 82 + 6 2 + 8 2 82 + 62 − 8
x = =9 and y = = 1.
2 2
Therefore x = ±3 and y = ±1. Since x · y = 3 > 0, then, x and y have the same
sign. Hence
case 1: when x = 3, we have y = 1. In this case z1 = 3 + i
case 2: when x = −3, the real number y = −1 then z2 = −3 − i.
2. We define z = x + i y such that z 2 = 3 − 4 i. Then, z 2 = (x + i y)2 = 3 − 4 i.
This involves x2 − y 2 = 3, 2xy = −4 and x2 + y 2 = 5. Thus we obtain
√ √
2 32 + 42 + 3 2 32 + 4 2 − 3
x = =4 y = = 1.
2 2
That is x = ±2 and y = ±1. Now we use the sign of xy = −2 < 0 to determine x
and y.
1.4. SECOND ORDER POLYNOMIAL EQUATIONS 19
− 1 + 3 i, 2 + i, 7 + 5 i, 4 − 2 i, 8 − 7 i, −2 + i, 4 − 9 i, −1 − i, 3 i.
−b
z= ·
2a
b+α b−α
z1 = − or z2 = − .
2a 2a
i z 2 + (1 + i)z + 2 + 3 i = 0, 3z 2 + 5 iz + 3 + 2 i = 0, z 2 + (2 + 3i)z + 1 = 0.
Fix some natural n ≥ 3. For every complex number z we are woundering if these
equations z n = 1 and z n = a + i b have solutions.
In the next section we will tackle the previous equations. We just precise that,
the solutions of the first one are called the nth roots of the unit and the solutions
of the second one are called nth roots of the complex number z.
z n = rn ei n θ = 1.
3. The points Mk form a regular polygon that is inscribed in the circle of center
O and radius 1.
9 √ 9 √
q q
θ + 2i πk π/4 + 2i πk
zk = 2 exp i = 2 exp i ,
9 9
Lemma 1.6.3. If z is a complex number that satisfies |z| = 1 then, there exists
a real number θ, such that,
z = exp(i θ).
1.7. EXERCISE SESSION 23
z = cos(θ) + i sin(θ).
Exercise 1.7.1. In the complex plane, we define i as the complex number that
has modulus 1 and argument π2 ·
6
1. Prove that 1 + i = −8i.
z 2 − z12 = 0 ⇔ z − z1 z + z1 = 0.
√ 6
6π
3π
3π
3π
z06 = 2 exp i = 8 exp i = 8 cos + i sin = −8i
4 2 2 2
b. Using the second point in the comment above, one finds that
2
z 2 − − 2 + 2i = z + (−2 + 2i) z − (−2 + 2i) = 0
p √ p √
Exercise 1.7.2. We consider the complex number z = − 2 + 2 + i 2 − 2.
1. Determine the algebraic form of z 2 .
2. Find the exponential form of z 2 .
Re(z) Im(z)
cos(θ) = and sin(θ) = ·
|z| |z|
p √ p √
Solution. We consider the complex number z = − 2 + 2 + i 2 − 2.
1. Using the first point of the comments we obtain
√ √ 2
q q
2
z = − 2+ 2+i 2− 2
√ √ q √ √ √ √
= 2 + 2 − 2 − 2 − 2 i 2 + 2 2 − 2 = 2 2 − 2i 2
2. From the algebraic from one can easily deduce that |z 2 | = 4 and arg(z 2 ) = − π4 ·
This implies that π
z 2 = 4 exp −i
4
3. The third point of the comments combines with 2 imply that
π π
z = 2 exp −i or z = −2 exp −i
8 8
Remarking that −1 = eiπ , we write −e−iπ/8 = ei(π−π/8) . Then, we have
π π
z = 2 exp −i or z = 2 exp i π −
8 8
π
As we know that cos(θ) is negative, we conclude that z = 2 exp i π − 8 ·
In this case we obtain
π π
z = 2 cos π − + i sin π − .
8 8
Exercise 1.7.3. Let U be the point associated to the complex number 1 and V
the point associated to i. We consider a complex number z 6= 0 and define the
complex
1
z0 = ·
z
1. Prove that arg(z 0 ) = arg(z) + 2kπ for some integer k.
26 CHAPTER 1. INTRODUCTION TO COMPLEX NUMBERS
2. Determine the set of all point M with affix z in the plane suh that
1
= z.
z
3. Establish that
z0 − 1
z−1
= −i ·
z0 − i z−i
4. Let M be the point associated to the complex number z. Prove that when M
belongs to the line (U V ), z−1
z−i is a real number.
0
5. Deduce from the previous questions arg zz0−1 −i ·
• arg(i) = π
2·
2
• For any complex number z, we have z · z = z .
• If ~u and ~v are two vectors, we define the oriented angle between ~u and ~v by
~u, ~v .
−−→ −→
• If AB (z−
−→ ) and AC (z−→ ) are two non-zero vectors then, one has
AB AC
!
z−→
AC
−→ −−→ −−→ −→
arg = arg z−→ − arg z−
−→ = ~u, AC − ~u, AB = AB, AC .
z−
−→ AC AB
AB
z0 − 1
z−1 z−1 z−1 1 z−1 z−1
= = = = · = −i ·
z0 − i iz − 1 iz + i2 i z+i i z−i z−i
4. In this question we assume that M (z) belongs to the line (U V ) and is different
to the points U and V . In this case, we can associate to the complex number
−−→ −−→
z − 1 the vector U M and V M to the complex z − i. One deduces that
z−1 −−→ −−→
arg = V M, UM .
z−i
−−→ −−→
Since M , U and V belong to the line (U V ), then, V M , U M = 0 + kπ, for
z−1
some integer k. Hence z−i is a real number.
28 CHAPTER 1. INTRODUCTION TO COMPLEX NUMBERS
1. z 2 + z + 1 = 0, 2. z 2 + 5z + 4 = 0, 3. z 2 + 6z + 9 = 0, 4. z 4 + 4z 2 + 3 = 0.
Comments. Here we remind the method to solve second order polynomial equa-
tions.
• To find the solution of the equation az 2 + bz + c = 0, where a 6= 0, we proceed
as follows: We compute the discriminant
∆ = b2 − 4ac
aZ 2 + bZ + c = 0.
Z 2 + 4Z + 3 = 0. (1.7.2)
z n = a + ib
• We have to determine
the exponential form of the complex number a+ib which
is |a + ib| exp i θ , where arg(a + ib) = θ + 2 iπ.
• θ = arg(a + ib)
• r = |a + ib|
• ω = arg(z)
• a + ib = r exp(iθ)
• z = |z| exp(i ω)
30 CHAPTER 1. INTRODUCTION TO COMPLEX NUMBERS
√ θ + 2kπ
|z| = n
r and ω= ·
n
Consequently, for every integer k, such that, 0 ≤ k ≤ n − 1, we have
√
θ + 2kπ
n
zk = r exp i ·
n
√ √
1. We consider the equation z 6 = 8. In this case we have |z| = 6
8= 2 and
θ = 0. We conclude that
√ √
2kπ kπ
zk = 2 exp i = 2 exp i ,
6 3
p
4
√ √
2. For the equation z 4 = 1 − i, we have |z| = 2 = 8 2 and θ = − π4 · Then,
for 0 ≤ k ≤ 3, we have
√
8 π kπ
zk = 2 exp i − + ·
16 2
√ √
3. Now we consider the equation z 7 = 1 − i 3. Then, |z| = 7 2 and θ = − π3 .
From this we deduce that
√
7 π 2kπ
zk = 2 exp i − + ,
21 7
for 0 ≤ k ≤ 6.
1.8. PROBLEMS 31
Comments. To find the solution of an equation of the form a3 z 3 +a2 z 2 +a1 z+a0 =
0, we proceed as follows:
• We find a particular solution z0 of the equation.
• We write this equation in the form (z − z0 )(az 2 + bz + c) = 0.
• We determine the solutions of the equation az 2 + bz + c = 0.
Solution. We consider the equation z 3 + 3z + 4z = 0. If we replace z by 0, we
obtain 0 + 3 · 0 + 4 · 0 = 0. Therefore z0 = 0 is a solution to the equation. Now we
have to write the equation in the form
z az 2 + bz + c = 0.
z(az 2 + bz + c) = az 3 + bz 2 + cz = z 3 + 3z 2 + 4z
z(z 2 + 3z + 4) = 0 ⇐⇒ z = 0 or z 2 + 3z + 4 = 0.
1.8 Problems
Problem 1.8.1. Let x and y be two real numbers. We define the complex number
z = x + i y. We suppose that z 6= 1 − 3i. and we consider the complex number
z + 3 + 7i
Z= ·
z − 1 + 3i
1. Determine the algebraic and the trigonometric form of Z.
2. Let M (x, y) be the point of the plane associated to z. Find the set of all points
M in the plane such that,
a. Z is a real number
32 CHAPTER 1. INTRODUCTION TO COMPLEX NUMBERS
1. (1 + i)z 2 − z − (1 − i) = 0, 2. 2z 2 − 2z + 5 = 0, 3. z 4 + 5z + 4 = 0,
√
4. z 4 − 2z 2 cos(θ) + 1 = 0, 5. z 3 = 3 − i.
√
Problem 1.8.7. Consider the complex number Z = (1 + i) 3−i .
1. Determine the algebraic and the trigonometric form of Z.
π π
2. Deduce from 1 the exact values of cos 12 and sin 12 ·
Problem 1.8.8. Let z be a complex number which is different to 1. We define the
complex number z 0 = 1 + z + z 2 + z 3 + z 4 .
1. Prove that
1 − z5
z0 = ·
1−z
2i π
, determine z 0
2. Taking z = exp 5
6. Deduce from the previous questions that cos π5 is a solution to the equation
4x2 − 2x − 1 = 0.
π
7. Solve the equation 4x2 − 2x − 1 = 0 and determine the exact value of cos 5 .
1.8. PROBLEMS 33
2. Now we take z = 1 + i.
a. Compute P (1 + i).
b. Deduce from 2.a) another solution to the equation P (z) = 0.
c. Find the solutions of the equation P (z) = 0.
Problem 1.8.10. Find a necessary and sufficient conditions on α and β, such that
1 + exp(i α) + exp(i β) = 0.
Problem 1.8.11. Define the trigonometric form of the following complex numbers
Problem 1.8.13. Let z belong to C. We define the function P (z) = z 3 +z 2 +2z −4.
1. Check that P (z) = (z − 1)(z 2 + 2z + 4).
2. Find the solutions of the following equation P (z) = 0.
34 CHAPTER 1. INTRODUCTION TO COMPLEX NUMBERS
CHAPTER 2
INTRODUCTION TO
SEQUENCES
Contents
2.1 Generalities on Sequences . . . . . . . . . . . . . . . . . 36
2.1.1 Computation of the Terms of a Sequence . . . . . . . . 37
2.1.2 Bounded Sequences . . . . . . . . . . . . . . . . . . . . 39
2.1.3 Monotonic Sequences . . . . . . . . . . . . . . . . . . . 39
2.2 Proof by Induction . . . . . . . . . . . . . . . . . . . . . 41
2.2.1 First Form of Induction . . . . . . . . . . . . . . . . . . 41
2.2.2 Second Form of Induction . . . . . . . . . . . . . . . . . 43
2.3 Study of Limits of Sequences . . . . . . . . . . . . . . . 44
2.3.1 Definitions of Limits of Sequences . . . . . . . . . . . . 44
2.3.2 General Theorems on Convergence . . . . . . . . . . . . 46
2.3.3 Elementary Operations on Limits . . . . . . . . . . . . . 53
2.4 Exercise Session . . . . . . . . . . . . . . . . . . . . . . . 59
2.5 Problems . . . . . . . . . . . . . . . . . . . . . . . . . . . 66
In this chapter we deal with functions with domain the set of natural numbers
N or a subset of N. These particular functions are called sequences. In other words
a sequence can be defined as a function which goes from N into R. That is, we have
f : N → R or f : D → R, with D ⊂ N. Usually we denote f (n) the image of n
by the function f . Since the set N is countable, the range of N which is denoted
by f (N) is also countable. In this case we can enumerate the elements of f N .
This allows to use the
notation fn instead of f (n). In this situation the set f (N) is
represented by fn n∈N .
We specify that the real number fn is called the nth term of the sequence
(fn )n∈N . The readers should make the difference between fn and (fn )n∈N . By
(fn )n∈N we mean all the terms of the sequence while fn designates the nth term
35
36 CHAPTER 2. INTRODUCTION TO SEQUENCES
un = f (n), (2.1.1)
We emphasize that the natural number n0 can be 0 if the first term of the
sequence is u0 . It can be 1 if the first term is u1 , · · · · We will specify later on what
we mean by the first term of a sequence. This definition means that the term un
depends directly on n.
un+1 = f (un ), ∀ n ≥ n0 .
2.1. GENERALITIES ON SEQUENCES 37
In this definition one can observe that the term un+1 depends on the previous one:
un . The relation of dependance is defined by the function f .
Definition 2.1.5. The initial term or the first term of a sequence (un ) is the
term that has the smallest index.
un0 = f (n0 ).
Let n1 > n0 be some positive integer. In order to compute un1 , we use the following
formula
un1 = f (n1 ).
• u0 = 2 · 0 + 3 = 3, • u1 = 2 · 1 + 3 = 5, • u2 = 2 · 2 + 3 = 7,
• u10 = 2 · 10 + 3 = 23, • u100 = 2 · 100 + 3 = 203.
Exercise 2.1.7. For each of the following sequences determine u0 , u2 , u10 and u25
1. ∀ n ∈ N, un = 3 n + 1, 2. ∀ n ∈ N, un = (2 n + 1)2 + 5,
1
3. ∀ n ∈ N, un = 3n+1 − 2n , 4. ∀ n ∈ N, un = ,
n+3
5. ∀ n ∈ N, un = n3 − 2 n + 1.
38 CHAPTER 2. INTRODUCTION TO SEQUENCES
∀ n ≥ 0, un+1 = 2 un + 1.
• u1 = 2 u0 + 1 = 2 · 2 + 1 = 5, • u2 = 2 u1 + 1 = 2 · 5 + 1 = 11,
• u3 = 2 u2 + 1 = 2 · 11 + 1 = 23, • u4 = 2 u3 + 1 = 2 · 23 + 1 = 47.
1 1
u1 1 1 u2 2 1
• u2 = = = , • u3 = = = 2 = ·
u1 + 1 1+1 2 u2 + 1 1 3 3
+1
2 2
Exercise 2.1.10. Compute the five first terms of each of the following sequences
u0 = 1 u1 = 0
1. 2.
∀n ≥ 0, un+1 = 3 un − 3, ∀n ≥ 1, un+1 = 2 u2n + 1,
u =1
0
3.
∀n ≥ 0, un+1 = 2 un + 1 ·
un
Definition 2.1.11. A sequence (un )n≥0 is bounded above , if there exists a real
number M such that, ∀ n ≥ 0,
un ≤ M.
un ≤ 1.
0 ≤ un = n2
∀ n ≥ 0, m ≤ un ≤ M (2.1.3)
un ≤ un+1 .
Example 2.1.18. Consider the sequence un = n2 , for all n ≥ 0. Then, the sequence
(un ) satisfies un = n2 ≤ (n + 1)2 = un+1 . Therefore it is increasing.
40 CHAPTER 2. INTRODUCTION TO SEQUENCES
Definition 2.1.19. We say that a sequence (un )n≥0 is decreasing, if for all n ≥ 0
we have,
un ≥ un+1 .
Proof. The proof of this theorem is quite simple. If forall natural number n ≥ 0,
un+1 − un ≥ 0, then we have un+1 ≥ un . Therefore un n≥0 is increasing.
On the other hand if for everynatural number n,we have un+1 − un ≤ 0, then,
un+1 ≤ un . We conclude that un n≥0 is decreasing.
Remark 2.1.22. When the inequalities are strict we say that the sequence is strictly
increasing or strictly decreasing.
Example 2.1.23. We define the following sequence
u0 = 2
3.
∀ n ≥ 0, un+1 = un + 3 n + 2.
5
3. ∀ n ≥ 1, un = 7 + ·
n2
Let n0 be a natural number. We consider the property P (n) for any natural
number n ≥ n0 . We would like to show that for each n ≥ n0 the property P (n) is
true. But we know that the set n ∈ N ; n ≥ n0 is infinite. It seems to be a waste
of time to want to check P (n) for any n ≥ n0 . Nevertheless, proceeding intelligently
we can prove that P (n) is true for any n ≥ n0 . Our strategy will runs in this way,
2.2. PROOF BY INDUCTION 41
n (n + 1)
1 + 2 + 3 + ... + n = ·
2
We define
n (n + 1)
P (n) : ∀ n ≥ 1, 1 + 2 + 3 + ... + n = ·
2
To establish P (n) for n ≥ 1, we proceed in three steps.
1 (1+1)
Step 1: We consider n = 1, then , 1 = 2 = 1. Therefore P (1) is true.
42 CHAPTER 2. INTRODUCTION TO SEQUENCES
n2 (n+1)2
2. P (n) : ∀ n ≥ 1, 1 + 8 + 27 + ... + n3 = 4 ·
n (n+1)(6 n3 +9 n2 +n−1)
3. P (n) : ∀ n ≥ 1, 1 + 16 + 81 + ... + n4 = 30 ·
Step 2: We assume that the property holds for any positive integer less than k for
some natural number k. That is for every k 0 such that 0 ≤ k 0 ≤ k we have
0 ≤ uk0 ≤ 2.
44 CHAPTER 2. INTRODUCTION TO SEQUENCES
∀ n ≥ 0, un+1 = 4 un − 1.
5
Establish that for all n ≥ 0, un = 3 4n + 31 ·
If there is no risk of ambiguity we will adopt from now on, the following no-
tations: we will write (un ) instead of (un )n≥n0 or (un )n≥0 , in order to simplify
notations. But when the situation seems to be confused, for the sake of clarity we
will specify the indices. Moreover throughout this chapter we will be concerned by
real sequences.
Let (un ) be a sequence. As (un ) is a function, we can be attempted to study
the behaviour of (un ) when n approaches +∞. Since we cannot reach +∞, in this
case we say that we are looking for the limit of the sequence at +∞. Below, we aim
to specify the nature of (un ) in accordance with the value of the limit.
We notice that the limit of a sequence holds only at +∞.
un − ` ≤ ε.
This definion can be rewriten as follows: there exists N ∈ N such that, for any
n ≥ N the term un belongs to a neighborhood of `. We interpret this by saying
that for any n ≥ N the term un is approximately equal to `. Using the absolute
value we write
lim |un − `| = 0.
n→+∞
Definition 2.3.2. We say that the sequence (un ) converges to the real number
`, if
lim un = `.
n→+∞
In this case the sequence (un ) is called a convergent sequence. If the limit ` is
+∞ or −∞, we say that (un ) is a divergent sequence. We will see below another
characterization of divergent sequences.
1
Example 2.3.3. For all n ≥ 1, we define the sequence un = n· Then, we have
1
lim un = lim = 0.
n→+∞ n→+∞ n
Therefore, the sequence (un ) converges to 0.
The following example shows a sequence which is divergent.
Example 2.3.4. We consider the sequence un = 2 n, for every n ≥ 0. One sees
that
lim un = lim 2 n = +∞.
n→+∞ n→+∞
Hence, the sequence (un ) is divergent.
Definition 2.3.5. A sequence (un ) converges to +∞, if for any real number
A > 0, there exists some natural number n, such that,
un > A.
Remark 2.3.6. This definition means that the sequence (un ) cannot be bounded
above.
Example 2.3.7. We take the sequence un = n2 , ∀ n ≥ 0. Then
lim n2 = +∞.
n→+∞
Definition 2.3.8. A sequence (un ) converges to −∞, if for any real number A,
one can find some natural number n such that,
un < A, ∀ n ≥ N.
Theorem 2.3.11. If `1 and `2 are two real numbers and (un ) a sequence such
that
lim un = `1 and lim un = `2 ,
n→+∞ n→+∞
then, `1 = `2 .
Remark 2.3.12. This theorem clarifies the fact that any convergent sequence has
a unique limit.
Proof. Let (un ) be a convergent sequence. We assume
Since (un ) is convergent we deduce that `1 and `2 are finite. Therefore, we have
Consequently we obtain `1 = `2 .
As a consequence of this lemma, we have the following corollary.
Corollary 2.3.13. Any real sequence (un ) that has more than one limit is di-
vergent.
wn = u2n+1 = −1
It is obvious that
From this we observe that the sequence (un ) has two different limits. Then, the
previous corollary implies (un ) is diverges.
Theorem 2.3.15. If (un ) and (vn ) be two real sequences and N some natural
number such that for every n ≥ N , un ≤ vn then
lim un ≤ lim vn .
n→+∞ n→+∞
2.3. STUDY OF LIMITS OF SEQUENCES 47
Proof. We consider two real sequences (un ) and (vn ) and we suppose that there
exists a natural number N , such that, for avery n ≥ N , we have un ≤ vn . We define
wn = un − vn and we assume that
lim wn = ` > 0.
n→+∞
The fact that (wn ) converges to a strictly positive real number ` means that
for some n large enough (n → +∞) we have wn > 0. That is, there exists a
natural number N0 ≥ N such that for every n ≥ N0 one has wn > 0. This involves
un − vn > 0, which is absurd. Hence, we have
Corollary 2.3.16. If (un ) is a real sequence, and N some natural number, such
that, for every n ≥ N , 0 ≤ un , then,
0 ≤ lim un .
n→+∞
In the corollary of theorem 2.3.11 we have shown that a sequence which has
two different limits diverges. The following theorem, will show that a divergent
sequence can have a unique limit. In this case this limit is equal to +∞ or −∞.
Theorem 2.3.17. Any increasing sequence which is not bounded above converges
to +∞.
Proof. We consider an increasing sequence (un ) which is not bounded above. Hence,
for all real number A > 0, we can find a natural number N , such that, uN > A.
Since (un ) is an increasing sequence, we deduce that for all n ≥ N , un > A.
Therefore using definition 2.3.5 we have
lim un = +∞.
n→+∞
Theorem 2.3.18. Any decreasing sequence which is not bounded below converges
to −∞.
Proof. We take a decreasing sequence (un ). We assume that (un ) is not bounded
below. Therefore for any real number B < 0 there exists N ∈ N such that uN < B.
Since (un ) is a decreasing sequence, we have un < B, for all n ≥ N. From definition
2.3.8 we deduce that
lim un = −∞.
n→+∞
48 CHAPTER 2. INTRODUCTION TO SEQUENCES
In many situations, we are not able to give the exact value of the limit when
a sequence converges. In these cases the best option which offers to us is just
to say that the sequence converges without specifying its limit. The monotonic
convergence theorem is one of such result that will allow us to say that a sequence
is convergent without giving its limit.
1. Establish that ∀ n ≥ 0, 0 ≤ un ≤ 2.
2. Prove that (un ) is an increasing sequence.
3. Is the sequence (un ) convergent ?
Solution. We consider the previous sequence and we aim to prove that P (n) : ∀ n ≥
0, 0 ≤ un ≤ 2
1. To this end we proceed by indution
Step 1: We fix n = 0 then we have 0 < u0 = 1 < 2. This implies P (0).
Step 2: We assume now that the property holds for all positive integer less
than or equal to some natural number k. That is, for every k 0 such that
k 0 ≤ k, one has 0 ≤ uk0 ≤ 2.
Step 3: Now we have to prove P (k + 1). In other words we want to establish
that
0 ≤ uk+1 ≤ 2.
Since 0 ≤ uk ≤ 2, we have
√ √
2 ≤ 2 + uk ≤ 4 ⇒ 0 ≤ 2≤ 2 + uk ≤ 2.
Then the property is true for k + 1
2.3. STUDY OF LIMITS OF SEQUENCES 49
1. Prove ∀n ≥ 0, 1 ≤ un
2. Study the variations of the sequence (un ).
3. Deduce from the two previous questions that (un ) converges.
Solution. 1. We define the property P (n) : ∀n ≥ 0, 1 ≤ un . To prove P (n) we
proceed in this way.
Step 1: We set n = 0. Then, we have u0 = 2 > 1. Then, P (0) holds.
Step 2: We consider some natural number k and we assume that the property
is satisfied by any positive integer k 0 ≤ k. Therefore, ∀ k 0 such that
k 0 ≤ k, we have 1 ≤ uk0 .
Step 3: Now we have to prove that 1 ≤ uk+1 . We recall that we assumed in
the previous step that for all k 0 ≤ k, uk0 ≥ 1. Then 1 ≤ uk . Multiplying
by 2 in the previous inequality, one gets 2 ≤ 2 uk . Thus one obtains
1 ≤ 2 uk − 1. Considering the square root in the previous inequality, we
obtain √
1 ≤ 2 uk − 1 = uk+1 .
Therefore P (k + 1) is true.
Conclusion: For any n ≥ 0, un ≥ 1.
2. We know that to study the variations of a sequence, we have to find the sign
of un+1 − un . For any n we have
√ −u2n + 2 un − 1 −(un − 1)2
un+1 − un = 2 un − 1 − un = √ = √ ≤ 0·
un + 2 un − 1 un + 2 un − 1
50 CHAPTER 2. INTRODUCTION TO SEQUENCES
This provides
un+1 − un ≤ 0.
Therefore (un ) is a decreasing sequence.
Exercise 2.3.22. Let (un )n≥0 and (vn )n≥0 be two real sequences, such that, u0 > 0,
v0 > 0. For every natural number n ≥ 1 we define
√
un+1 = un vn
vn+1 = un + vn ·
2
1. Establish that for all n ≥ 1, un ≤ vn .
3. Deduce from the two previous questions that (un ) and (vn ) are convergent
sequences and they have the same limit.
M − ε ≤ uN ≤ un ≤ M.
2.3. STUDY OF LIMITS OF SEQUENCES 51
On the other hand we have M + ε ≥ M for any ε > 0. This yields the following
property: for all ε > 0, there exist a natural number N , such that, for avery n ≥ N ,
we have
M − ε ≤ un ≤ M + ε.
This can be rewritten in this way:for all ε > 0, there exist a natural number N ,
such that, for avery n ≥ N , un − M ≤ ε. This prove that
lim un = sup un .
n→+∞ n≥0
Proof. Take the decreasing and bounded below sequence (un ). Then, the real num-
ber m := inf n≥0 un is defined and for every n ≥ 0, m ≤ un . Let ε > 0. There exists
some natural number N , such that, m ≤ uN ≤ m + ε. As the sequence (un ) is
decreasing, we have for all n ≥ N ,
m ≤ un ≤ uN ≤ m + ε.
This implies that, for every ε > 0, there exists a natural number N , such that for
every n ≥ N , m − ε ≤ un ≤ m + ε. From this we deduce that for every ε > 0, there
exists a natural number N , such that for every n ≥ N , we have un − m ≤ ε. Hence
we have
lim un = m = inf un .
n→+∞ n≥0
When the sequence (un ) is only bounded, and there is no information on its
variations, we cannot say that the sequence is convergent. But what we know is we
can extract a convergent sequence from (un ).
Theorem 2.3.27 (Sandwich Theorem). If (un ), (vn ) and (wn ) are three se-
quences such that vn ≤ un ≤ wn for every n ≥ N for some natural number N.
Moreover we suppose that
lim vn = lim wn = `,
n→+∞ n→+∞
then,
lim un = `.
n→+∞
52 CHAPTER 2. INTRODUCTION TO SEQUENCES
Proof. We consider three sequences (un ), (vn ) and (wn ), such that, vn ≤ un ≤
wn for every n ≥ N for some natural number N. Moreover we assume that,
limn→+∞ vn = limn→+∞ wn = `. Therefore we have
` ≤ lim un ≤ `.
n→+∞
That is,
lim un = `.
n→+∞
cos(n)
un =
n
∀n ≥ 1 we use the properties of the function cosinus. For all n ≥ 1, −1 ≤ cos(n) ≤
1. This means that
−1 cos(n) 1
≤ ≤ ·
n n n
−1
As we know that, limn→+∞ = limn→+∞ n1 = 0, we conclude that
n
cos(n)
lim = 0·
n→+∞ n
Exercise 2.3.29. Find the limits of the following sequences:
Proof. Take the sequence (un ) which converges to `. There exists a natural number
N such that for every n ≥ N , un − ` is bounded by 1. In other words, |un − `| < 1
for n ≥ N. Let n be a natural number, such that, n ≥ N then we have
Then, for every n ∈ N, we have |un | ≤ M. Therefore, the sequence (un ) is bounded.
4n+1 +1
Example 2.3.32. To prove that the sequence vn = 5n is bounded we show that
4n+1 + 1
lim vn = lim = 0.
n→+∞ n→+∞ 5n
We deduce that the sequence (vn ) is convergent. Hence it is bounded.
Since we defined the limit of a sequence, we want to know if the set of convergent
sequences is stable by the elementary algebraic operations.
then,
lim (un + vn ) = lim un + lim vn = ` + `0 .
n→+∞ n→+∞ n→+∞
Given ε > 0, there exist two real numbers N1 and N2 , such that, for every n ≥ N1 ,
|un − `| ≤ 2ε and for every n ≥ N2 , |vn − `0 | ≤ 2ε .
Considering N = max(N1 , N2 ), and n ≥ N we have
ε ε
|un − `| ≤ and |vn − `0 | ≤ ·
2 2
Hence
|un + vn − (` + `0 )| = |(un − `) + (vn − `0 )| ≤ |un − `| + |vn − `0 | ≤ ε.
We conclude that
lim un + vn = ` + `0 .
n→+∞
This completes the proof.
lim un = `,
n→+∞
lim λun = λ `.
n→+∞
54 CHAPTER 2. INTRODUCTION TO SEQUENCES
Proof. To prove this proposition, we consider a real convergent sequence (un ), such
that,
lim un = `.
n→+∞
Let λ ∈ R. If λ = 0 the result holds.
Now we assume that λ 6= 0. The convergence of (un ) to ` implies that, for all
ε > 0, there exist some N ∈ N, such that, for every n ≥ N , we have
ε
|un − `| ≤ .
|λ|
Now we consider a natural n ≥ N. Straightforward computations show that
ε
λ un − λ ` = |λ| |un − `| ≤ |λ| = ε.
|λ|
Therefore,
lim λ un = λ `.
n→+∞
The proposition is proved.
Now, we want to investigate the limit of the product of two convergent sequences.
Proposition 2.3.35. If (un ) and (vn ) are tw real sequences, such that,
then,
lim (un · vn ) = lim un · lim vn = ` · `0 .
n→+∞ n→+∞ n→+∞
Given ε > 0, we can find two natural numbers N1 and N2 , such that,
for every n ≥ N1 , |un − `| ≤ ε
for every n ≥ N2 , |vn − `0 | ≤ ε·
From this we deduce that for every n ≥ N = max(N1 , N2 ), we have
|un vn − ` `0 | ≤ |un − `| |vn − `0 | + |`| |vn − `0 | + |`0 | |un − `| ≤ ε2 + |`|ε + |`0 |ε.
Consequently, limn→+∞ un vn = ` `0 .
Remark 2.3.36. We insist on the fact that (un ) and (vn ) should be convergent, if
we want to be sure to get a limit for un + vn or for vn · vn . Otherwise the existence
of the limit of un + vn or un vn is not guaranteed.
Such pathological cases will be investigated below. From the previous propo-
sition readers can easily conclude that the set of convergent sequences has the
property of a vector space.
In the theorem below we study the cases when ` and `0 are not necessarily real
numbers. They can be also +∞ or −∞.
2.3. STUDY OF LIMITS OF SEQUENCES 55
Theorem 2.3.37. Let (un ) and (vn ) be two real sequences, such that,
lim un + vn = ` + `0
n→+∞
lim un + vn = ` ± ∞ = ±∞
n→+∞
lim un + vn = ±∞ + `0 = ±∞
n→+∞
lim un + vn = +∞
n→+∞
lim un + vn = −∞
n→+∞
lim un + vn = +∞ − ∞ = undefined
n→+∞
lim un + vn = −∞ + ∞ = undefined.
n→+∞
Proposition 2.3.38. Let (un ) and (vn ) be two sequences and ` a real number,
such that,
lim un = ` and lim vn = +∞.
n→+∞ n→+∞
Then,
lim un · vn = +∞ if `>0
n→+∞
lim un · vn = −∞ if `<0
n→+∞
lim un · vn = undefined if ` = 0.
n→+∞
56 CHAPTER 2. INTRODUCTION TO SEQUENCES
Proposition 2.3.39. Let (un ) and (vn ) be two sequences and ` a real number,
such that,
lim un = ` and lim vn = −∞.
n→+∞ n→+∞
Then,
lim un · vn = −∞ if `>0
n→+∞
lim un · vn = +∞ if `<0
n→+∞
lim un · vn = undefined if ` = 0.
n→+∞
Proposition 2.3.40. Let (un ) and (vn ) be two sequences, such that,
Then,
lim un · vn = ±∞.
n→+∞
lim un = `.
n→+∞
Then, there exists some natural number N , such that, for every n ≥ N ,
|un | > 0.
Proof. Since (un ) converges to `, for every ε > 0, there exists a natural number N0
such that for every n ≥ N0 , |un − `| ≤ ε. In particular if we fix ε = 21 `, we can find
some natural number N1 , such that, for every n ≥ N1 , |un − `| < 21 |`|. Let n be a
natural number greatter than N1 , then, we have
1 1
|`| = |(` − un ) + un | ≤ |un − `| + |un | < |`| + |un | =⇒ |`| < |un |.
2 2
This proves the proposition.
Using this proposition we can prove the following theorem.
2.3. STUDY OF LIMITS OF SEQUENCES 57
Proof. We take a real sequence (un ) and we consider a real number ` 6= 0, such
that,
lim un = `.
n→+∞
Using Proposition 2.3.41 , we can find some natural number N0 , such that for every
natural number n greater than or equal to N0 , we have, 12 |`| < |un |.
The fact that (un ) converges to `, means that for every ε > 0, there exists
N1 ∈ N, such that, for every n ≥ N1 ,
1 2
|un − `| ≤ |`| ε.
2
Now, we take a natural number n ≥ max(N0 , N1 ). One has
1 1 un − ` |un − `| 1
− = = = · |un − `|·
un ` un ` |un | |`| |un | |`|
1 1 2 |`|2
− ≤ 2 ε = ε·
un ` |`| 2
Theorem 2.3.43. Let (un ) and (vn ) be two real sequences. We consider ` ∈ R
and `0 ∈ R∗ ,such that,
Then,
un `
lim = 0· (2.3.2)
n→+∞ vn `
We specify that in the preceding theorems we assumed that the limit of the
sequence which is in the denominator is a nonzero real number and the limit of the
numerator is a real number. This simplify a lot the computations of limits. In the
case where ` and `0 can be infinite the study of limit of division of sequences is more
complicate. To give an overview of this difficulty we state the following theorem.
58 CHAPTER 2. INTRODUCTION TO SEQUENCES
Theorem 2.3.44. Let (un ) and (vn ) be two real sequences, ` and `0 two real numbers
or ±∞, such that,
lim un = ` and lim vn = `0 .
n→+∞ n→+∞
We have also
lim un − vn = undefined.
n→+∞
we obtain 2 1
lim − 1 + + 2 = −1, and lim n = +∞.
n→+∞ n n n→+∞
Exercise 2.4.1. For every natural number n, we define the sequence (un )
1
u0 =
2
√
un+1 = un , ∀ n ≥ 0.
Comments. If we deal with sequences, we should be careful with the initial term
( first term ). Because the initial term is not necessarily the term with index 1. It
depends on the context. The following recommandations can be useful
• To prove any proposition with sequences, think in term of induction.
• To study variations of a sequence (un ) you should define the sign of
un+1 − un .
60 CHAPTER 2. INTRODUCTION TO SEQUENCES
√ un − u2n un (1 − un )
un+1 − un = un − un = √ = √ ·
un + un un + un
√
As we know that 12 ≤ un ≤ 1 one deduces that un 1 − un ≥ 0 and un + un ≥ 0.
Exercise
√ 2.4.2. The aim of this exercise is to define the positive real number
2. To this end we define the real sequence (un )
u0 = 1,
1 2
u = u + , ∀ n ≥ 0.
n+1
n
2 un
√
1. Show that for any n ≥ 1, we have 2 ≤ un .
2. Establish that (un )n≥1 is a decreasing sequence and deduce from this, that
for every n ≥ 1
√ 3 √
un + 2 ≤ + 2.
2
3. Prove that for any n ≥ 0, we have
√ √ !2n
un − 2 1− 2
√ = √ .
un + 2 1+ 2
Comments. The comments are the same are in the previous exercise.
un+1 − un .
lim q n = 0.
n→+∞
Solution. We consider
√ the sequence (un ).
1. To prove that 2 ≤ un , for every n ≥ 1, we proceed√ as follows: at first we fix
n = 1. In this case, we have u1 = 12 1 + 2 = 32 · Then, 2 ≤ u1 .
√
Secondly we assume that there exists some natural number k, such that, 2 ≤ uk .
Cosequently, we have
√ √ 2
√ √ u2k − 2 2uk + 2 uk − 2
1 2
uk+1 − 2= uk + − 2= = ≥ 0.
2 uk 2 uk 2 uk
√ √
This means that uk+1 ≥ 2. We conclude that for any n ≥ 1 we have 2 ≤ un .
2. For any natural number n ≥ 1, we have
1 2 2 − u2n
un+1 − un = un + − un = ≤ 0.
2 un 2 un
62 CHAPTER 2. INTRODUCTION TO SEQUENCES
√
Because 2 ≤ un for any n ≥ 1. As we have established that (un ) is decreasing for
every n ≥ 1, one deduces that un ≤ u1 = 32 . This implies that
√ 3 √
un + 2≤ + 2
2
3. Let n be a natural. It is obvious that when n = 0
√ √
u0 − 2 1− 2
√ = √ ·
u0 + 2 1+ 2
Then P (0) is true.
Now, we assume that the property holds for some natural number k. In this case,
we have
√ √ !2k
uk − 2 1− 2
√ = √ ·
uk + 2 1+ 2
The next step consist of proving that the property is true for k + 1. Indeed, we have
√ √ √ √ √ 2
√
1 2
uk+1 − 2 2 uk + uk − 2 u k u k − 2 − 2 uk − 2 uk − 2
√ = √ = √ √ √ = √ 2
uk+1 + 2 1
uk + 2 + 2 uk uk + 2 + 2 uk + 2 u + 2
2 uk k
un+1 = 1 un (20 − un ) ,
∀ n ≥ 0.
10
2.4. EXERCISE SESSION 63
Comments. To study a sequence in the form un+1 = f (u) for some real function
f which is not affine, we to follow these steps.
• We have to determine the fixed points of f. We recall that a fixed point x0 is
a real number in the domain of f that satisfies f (x0 ) = x0 .
• We study the variations of the function f
• We identify the intervals which are stable
by f. In other words we have to find
the intervals [a , b] such that f [a , b] ⊂ [a , b].
• If the sequence admits a limit it should be a fixed point of f.
• To study the variations of (un ) we can use the variations of the function f
We remind also that the limit of a sequence which we know the variations, is given
by the monotonic convergence theorem. To prove any property of sequences use a
proof by induction.
Solution. Let x be in [0 , 20]. We define the function
1
f (x) = x 20 − x .
10
The function f is differentiable and f 0 (x) = 2 − 15 x. Therefore f is increasing in
[0 , 10] and decreasing in [10 , 20]. Moreover we observe that f (0) = 0 and f (10) =
10. The real numbers 0 and 10 are the fixed points of f. One can easily check that
f [0 , 10] = [0 , 10].
Then the interval [0 , 10] is stable by f. This means that if 0 ≤ x ≤ 10, then,
0 ≤ f (x) ≤ 10.
1. Let n be a natural number to prove that 0 ≤ un ≤ un+1 ≤ 10, we proceed in two
steps
Step 1: we prove that for every n ≥ 0, 0 ≤ un ≤ 10.
If we fix n = 0, we obtain 0 < u0 = 1 < 10. The property is true for 0.
Now we assume that 0 ≤ uk ≤ 10 for some natural number k. Here we can
see that
1
uk+1 = uk 20 − uk ≥ 0.
10
Because 0 ≤ uk ≤ 10, which implies that 20 − uk ≥ 0. On the other hand, we
have
1 1 2 1 2
uk+1 −10 = uk 20−uk −10 = − uk −20uk +100 = − uk −10 ≤ 0.
10 10 10
This means that uk+1 ≤ 10.
We conclude that for every natural number n, we have 0 ≤ un ≤ 10.
64 CHAPTER 2. INTRODUCTION TO SEQUENCES
f (uk−1 ) ≤ f (uk ).
This is due to the fact that f is increasing in [0 , 10]. The definition of the
sequence (un ) implies that uk ≤ uk+1 .
Combining step 1and step 2 we conclude that 0 ≤ un ≤ un+1 ≤ 10, for any natural
number n.
2. Since (un ) is increasing and bounded above, then, the monotonic convergence
theorem implies that (un ) converges. To determine the limit of (un ), we emphasize
that the limit ` satisfies
This leads to the identity f (`) = `. The real number ` is a fixed point. In this case
there are two possibilities ` = 0 or ` = 10. We claim that ` cannot be 0, because
1 ≤ un for any n ≥ 0 and (un ) is increasing. Finally we have ` = 10.
Exercise 2.4.4. Let (un ) be the sequence, such that, u0 = 13 and for any n ≥ 0,
1 4
un+1 = un + ·
5 5
We associate to (un ) the sequence (sn ) defined by
n
X
sn = uk = u0 + u1 + · · · + un .
k=0
12
1. Prove that for every n ≥ 0, we have un = 1 + 5n ·
Comments. Let (un ) be a real sequence. To establish any properties think at first
in terms of induction.
If q is any real number then,
n
X 1 − q n+1
1 + q + · · · + qn = qk = ·
1−q
k=0
2.4. EXERCISE SESSION 65
lim un = 1.
n→+∞
2. Deduce from 1 that the sequence (un ) converges and determine its limit.
66 CHAPTER 2. INTRODUCTION TO SEQUENCES
Solution. 1. Let x be a real number which belongs to [0 , 14]. We define the real
function
f (x) = 1.4 x − 0.05 x2 .
The function f is differentiable and f 0 (x) = 1.4 − 0.10x. This implies that the
function f is increasing in [0 , 14]. We have
f (0) = 0, f (8) = 8 and f [0 , 8] ⊂ [0 , 8].
Thus, we can say that 0 and 8 are fixed points and the interval [0 , 8] is stable by f.
Now, we take n = 0, one has 0 < u0 < u1 < 8. The property is true for 0.
If we assume that there exist k ∈ N∗ such that, 0 ≤ uk−1 ≤ uk ≤ 8, then,
2.5 Problems
√
Problem 2.5.1. The aim of this exercise is to define the positive real number 2.
To this end we define the real sequence (un )
u0 = 1,
1 2
un+1 = un + , ∀ n ≥ 0.
2 un
5
u0 = ,
2
1 5
, ∀ n ≥ 0.
un+1 =
un +
2 un
xn+1 = 1 xn + n − 2,
∀ n ≥ 0.
3
1. Compute x1 , x2 and x3 .
2. Prove that for every n ≥ 4, xn ≥ 0.
3. Deduce from 2. that for every n ≥ 5 , we have xn ≥ n − 3.
4. Find the limit of (xn )
Problem 2.5.8. We consider the sequence (tn ) defined by
t0 = 1,
tn+1 = tn − ln t2n + 1 , ∀ n ≥ 0.
Establish that
1
lim un = and lim vn = ea .
n→+∞ e n→+∞
u0 = a
u = 3
0
1. 2. un
1 un+1 = r + un − p , ∀n ≥ 0
u
n+1
= √ , ∀ n ≥ 0,
1 + u2n
2 − un
and r ∈ ]0 , 1[.
u ∈R
u 0 ∈ R+
0
1. 2.
p
un+1 = a un − 1
un+1 = (n + 1) + un , ∀ n ≥ 0, , ∀ n ≥ 0.
n+1
Problem 2.5.14. We consider the sequence un n≥1
defined by
2n
X 1 1 1 1
un = = + + ··· + · (2.5.1)
k n n+1 2n
k=n
un+1 = 1 + un ,
∀ n ≥ 0,
1 + en
1. Show that for every n ∈ N, we have 0 ≤ un ≤ α, where α is a real number
which satisfies
1+α
α= ·
1 + eα
70 CHAPTER 2. INTRODUCTION TO SEQUENCES
2. Deduce from 1. that (un ) converges and its limit verifies the following identity
1+`
`= ·
1 + e`
Contents
3.1 Arithmetic Sequences . . . . . . . . . . . . . . . . . . . . 71
3.1.1 Variations of an Arithmetic Sequence . . . . . . . . . . 73
3.1.2 Explicit Expression of an Arithmetic Sequence . . . . . 74
3.1.3 Sum of the n First Terms of an Arithmetic Sequence . . 76
3.2 Geometric Sequences . . . . . . . . . . . . . . . . . . . . 78
3.2.1 Explicit Expression of a Geometric Sequence . . . . . . 81
3.2.2 Sum of the n First Terms of a Geometric Sequence . . . 83
3.2.3 Limits of Geometric Sequences . . . . . . . . . . . . . . 85
3.3 Arithmetico-geometric sequences . . . . . . . . . . . . . 86
3.3.1 Limits of Arithmetico-Geometric Sequences . . . . . . . 89
3.4 Exercise Session . . . . . . . . . . . . . . . . . . . . . . . 90
3.5 Problems . . . . . . . . . . . . . . . . . . . . . . . . . . . 96
71
72 CHAPTER 3. ARITHMETIC AND GEOMETRIC SEQUENCES
Definition 3.1.1. We say that (un )n≥n0 is an arithmetic sequence with initial
term un0 and common difference d, if it is in the form
un0 ∈ R,
un+1 = un + d, ∀ n ≥ n0 .
un+1 = un + 3, ∀ n ≥ 0.
The sequence (un ) is arithmetic with initial term u0 = 2, and common difference
d = 3.
Now, we consider a general sequence (un )n≥n0 . According to the definition above
to establish that (un ) is an arithmetic sequence we have to proceed in this way
un+1 = un − 2, ∀ n ≥ 0.
un+1 − un = un − 2 − un = −2.
un = n + 2 n , for all n ≥ 0.
To establish whether (un ) is arithmetic, we point out that for any natural number
n we have
un+1 − un = (n + 1) + 2n+1 − n − 2n = 1 + 2n .
Since un+1 − un depends on n, it is not a constant real number. Hence (un ) is not
an arithmetic sequence.
3.1. ARITHMETIC SEQUENCES 73
un+1 − un = 3(n + 1) + 2 − 3n − 2 = 3n + 3 + 2 − 3n − 2 = 3.
Exercise 3.1.6. Establish whether the following sequences are arithmetic. In the
case they are arithmetic precise the initial term and the common difference.
u0 = 3 u0 = 1
1. 2.
un+1 = −2 + un , ∀ n ≥ 0, un+1 = 2 + un , ∀ n ≥ 0,
3. un = 8n−1 + 2n, ∀ n ≥ 0, 4. ∀ n ≥ 0, un = −n + 9.
un+1 − un = d.
Proof. Since (un ) is an arithmetic sequence with initial term un0 and common
difference d, we have, un+1 − un = d, for all n ≥ n0 . Therefore
• if d < 0, one obtains un+1 − un < 0 for all n ≥ n0 . Then (un ) is a decreasing
sequence
un+1 = un + 1.
In this example (un ) is an arithmetic sequence with initial term u0 = 1 and the
common difference d = 1 > 0. Then, (un ) is an increasing sequence.
Example 3.1.9. Let us consider the sequence:
u0 = 2
un+1 = un − 3.
5. ∀n ≥ 0, un = 3 n − 2.
un0 +1 = un0 + d,
un0 +2 = un0 +1 + d = un0 + 2,
un0 +3 = un0 +2 + d = un0 + 3 d.
Theorem 3.1.11. If (un ) is an arithmetic sequence with initial term un0 and
common difference d, then,
un = un0 + (n − n0 ) d, ∀ n ≥ n0 . (3.1.1)
Step 3: We need to prove that P(k + 1) is true. In other words we should prove
that uk+1 = un0 + (k − n0 + 1) d. We remind that (un ) is an arithmetic
sequences with common difference d. The recursive expresion of (un ) involves
un+1 = 3 + un , ∀ n ≥ 1.
un = u1 + (n − 1) d = 8 + 3 (n − 1) = 5 + 3 n.
Example 3.1.13. We consider the arithmetic sequence (vn ) with initial term v5 = 4
and common difference d = −6. Then,
v5 = 4
vn+1 = −6 + vn , ∀ n ≥ 5.
Corollary 3.1.14. For any arithmetic sequence (un ) with initial term u0 and
common difference d, we have
un = u0 + n d. (3.1.2)
The proof of this corollary is quite simple, Because we have just to take n0 = 0
in the proof of the theorem 3.1.11.
un+1 = un + 2, ∀ n ≥ 0.
In this example the initial term is u0 = 9 and the common difference d = 2. Using
corollary (3.1.2), we obtain un = u0 + n d = 9 + 2 n.
76 CHAPTER 3. ARITHMETIC AND GEOMETRIC SEQUENCES
Formulas (3.1.1) and (3.1.2) are interesting when there are some given terms
and we seek for the common difference of an arithmetic sequence. Indeed assume
that, we aim to define the arithmetic sequence (un ), such that, there are two given
terms un1 and un2 , with n2 > n1 . To define the expression of (un ) we have to find
the common difference. Thus we proceed as follows:
Step 1: Using theorem (3.1.1), we have un2 = un1 + (n2 − n1 ) d. This involves
un2 − un1
d= ·
n2 − n1
u8 = u3 + (8 − 3) d = 23 + 5 d = 7.
−16
This involves 5 d = −16. Therefore d = 5 . Using the value of d, we obtain
u3 = 23
16
un+1 = un − 5 , ∀ n ≥ 0.
Exercise 3.1.17. Let (un ) be an arithmetic sequence with initial term u5 = 8 and
common difference d = 3. Determine u100 , u1000 and u200 .
Exercise 3.1.18. We consider an arithmetic sequence (un ) which satisfies u2 = 10
and u10 = 12. Find the common difference, u5 , u7 and u20 .
Exercise 3.1.19. For every n ≥ 0 we define un = 7 − 5 n.
1. Find u0 , u1 and u2 and prove that (un ) is an arithmetic sequence.
2. Determine the common difference of (un ) and compute u11 .
Now we take an arithmetic sequene (un ) with initial term, u0 and common
difference d. We attempt to determine the sum u0 + u1 + · · · + un .
3.1. ARITHMETIC SEQUENCES 77
Proof. Let (un ) be an arithmetic sequence with initial term u0 , and common dif-
ference d. We have
u0 + u1 + · · · + un = u0 + u0 + d + · · · + u0 + n d
= (n + 1) u0 + d (1 + · · · + n)
n(n + 1) (n + 1) [u0 + (u0 + n d)]
= (n + 1) u0 + d = ·
2 2
Since un = u0 + n d, we obtain
(n + 1) (u0 + un )
u0 + u1 + · · · + un = ·
2
This proves the lemma.
Example 3.1.21. We take the arithmetic sequence
u0 = 3
un+1 = un + 2, ∀ n ≥ 0.
Theorem 3.1.22. Let (un ) be an arithmetic sequence with initial term up and
common difference d. Then,
n
X (n − p + 1) (up + un )
up + up+1 + · · · + un = uk = · (3.1.4)
2
k=p
To prove the theorem we use the same argument as in the preceding theorem.
Indeed, we have
up + up+1 + · · · + un = up + up + d + · · · + up + (n − p) d
= (n − p + 1) up + d (1 + 2 + · · · + (n − p))
(n − p + 1) (n − p) d
= (n − p + 1) up +
2
(n − p + 1) (up + up + (n − p) d) (n − p + 1) (up + un )
= = ·
2 2
78 CHAPTER 3. ARITHMETIC AND GEOMETRIC SEQUENCES
un+1 = 3 + un , ∀ n ≥ 5.
Definition 3.2.1. We say that (un )n≥n0 is a geometric sequence with initial
term un0 ∈ R, if there exists some real number q, such that, for every natural
number n ≥ n0 ,
un+1 = q un .
Definition 3.2.2. The real number q in the definition 3.2.1 is called the common
ratio of the sequence (un ).
un+1 = 3 un , ∀ n ≥ 0.
The sequence (un ) is geometric with initial term u0 = 2 and common ratio q = 3.
3.2. GEOMETRIC SEQUENCES 79
vn+1 = 2 vn , ∀ n ≥ 0.
We can see that the sequence (vn ) is geometric with initial term v0 = −1 and
common ratio q = 2.
Example 3.2.5. For every n ≥ 0, we set wn = 3n . Then, the sequence (wn ) is
geometric with initial term w0 = 1 and common ratio q = 3.
Indeed we have w0 = 30 = 1. Let n be a natural number, then,
wn+1 = 3n+1 = 3 × 3n .
un+1 = a un + b, ∀ n ≥ 0,
for some real numbers a ∈ R\{0 ; 1} and b ∈ R∗ . For every natural number n ≥ 0,
we define
b
vn = un − ·
1−a
Then, (vn ) is a geometric sequence.
To prove this , we take n ∈ N, and we assume that vn 6= 0. Then, we have
b b
vn+1 un+1 − 1−a aun + b − 1−a aun + b−ba−b
1−a
ba
aun − 1−a
= b
= b
= b
= b
vn un − 1−a un − 1−a un − 1−a un − 1−a
b
a un − 1−a
= b
= a·
un − 1−a
b
The sequence (vn ) is geometric with initial term u0 − 1−a and commom ratio a.
What we wanted to point out in this example is the long and complicated calcu-
lations on which this method can lead. The second inconvenience is, the fact that
80 CHAPTER 3. ARITHMETIC AND GEOMETRIC SEQUENCES
un+1 = 2 un + 1, ∀ n ≥ 0,
As we know that vn = un + 1, one has vn+1 = 2vn . Hence the sequence (vn ) is
geometric with initial term v0 = u0 + 1 = 2 + 1 = 3 and common ratio q = 2.
un+1 = 3 un − 4, ∀ n ≥ 0,
and we define the sequence vn = un − 2. Then we can prove that the sequence (vn )
is geometric.
Therefore vn+1 = 3vn . Consequently the sequence (vn ) is geometric with initial
term v0 = u0 − 2 = 1 − 2 = −1 and common ratio q = 3.
3.2. GEOMETRIC SEQUENCES 81
un+1 = q un , ∀ n ≥ 0.
u1 = q u0 ,
u2 = qu1 = q · qu0 = q 2 u0
u3 = qu2 = q · q 2 u0 = q 3 u0 .
Continuing this recursive procedure one can establish the following theorem
Theorem 3.2.9. If (un ) is a geometric sequence with initial term u0 and common
ratio q, then for every n ≥ 0,
un = u0 · q n . (3.2.1)
Proof. Let (un ) be a geometric sequence with initial term u0 and common ratio q.
We define the following proposition: P(n) : ∀ n ≥ 0, un = q n u0 . Our objective
now, is to prove P(n).
Step 2: We assume that the property holds for some natural number k. This
means that
uk = u0 · q k .
uk+1 = q × q k u0 = u0 · q k+1 .
un+1 = −2 un , ∀ n ≥ 0.
un+1 = 7 un , ∀ n ≥ 0.
up+1 = up · q,
up+2 = up+1 · q = up · q 2 ,
up+3 = up+2 · q = up · q 3 ,
up+4 = up+3 · q = up · q 4 .
Theorem 3.2.13. Let (un )n≥p be a geometric sequence with initial term up and
common ratio q. Then, ∀ n ≥ p,
un = up · q n−p . (3.2.2)
Proof. Let (un ) be a geometric sequence with initial term up and common ratio q.
To prove the theorem we need to check the following proposition:
P(n) : ∀ n ≥ p, un = up · q n−p .
un+1 = 5 un , ∀ n ≥ 3.
In this example, the initial term of the sequence is u3 = −2 and the common ratio
is q = 5. Using theorem 3.2.13, we obtain
un = u3 · q n−3 = −2 · 5n−3 .
un+1 = − 21 un , ∀ n ≥ 3.
Exercise 3.2.16. Determine the explicit form of each of the following sequences
w7 = 2
u1 = 1.5
1. 2.
wn+1 = wn , ∀ n ≥ 7,
un+1 = 2 un , ∀ n ≥ 1.
3 3
1 − q n+1
Tn = · (3.2.4)
1−q
Proof. We remind that this lemma can be proved by induction. But here we present
another method to prove it. We define
q Tn = q + q 2 + · · · q n + q n+1 .
84 CHAPTER 3. ARITHMETIC AND GEOMETRIC SEQUENCES
Hence, we have
(1 − q)Tn = Tn − q Tn = 1 + q + · · · + q n − q − q 2 − · · · − q n − q n+1
Xn Xn
=1+ qk − q k − q n+1 = 1 − q n+1 .
k=1 k=1
Proof. Let (un ) be a geometric sequence with initial term u0 and common ratio q.
Then,
u0 + u1 + · · · + un = u0 + u0 · q + · · · + u0 · q n = u0 1 + q + q 2 + · · · + q n
1 − q n+1
= u0 · Tn = u0 · ·
1−q
This proves the theorem.
Example 3.2.19. We consider the following geometric sequence (un )
u0 = 6
un+1 = 4 un , ∀ n ≥ 0.
To compute u0 + u1 + · + u21 we specify that the initial term of the sequence (un )
is u0 = 6 and the common ratio is q = 4. This provides
1 − q 21+1 1 − 422
S21 = u0 · =6· = 2(422 − 1).
1−q 1−4
Combining theorem 3.2.13 and lemma 3.2.17 we can prove the following general
theorem:
Theorem 3.2.20. For any geometric sequence (un ) with initial term up and
common ratio q 6= 1, we have
n
X 1 − q n−p+1
uk = up · · (3.2.6)
1−q
k=p
3.2. GEOMETRIC SEQUENCES 85
Proof. Since (un ) is a geometric sequence with initial term up and common ratio q
we have un = up · q n−p . This involves
1 − q n−p+1
up + up+1 + · · · + un = up (1 + q + · · · + q n−p ) = up · ·
1−q
un+1 = 3 un , ∀ n ≥ 2.
In this example the initial term is u2 = 2 and the common ratio is q = 3. Hence
one obtains
1 − 311−2+1 1 − 310
u2 + u3 + · · · + u11 = u2 × =2× = (310 − 1).
1−3 −2
Generally speaking the sum of the first terms of a geometric sequence is given
by the formula
1 − q number of terms
Sum=(initial term)× ·
1−q
Consider a geometric sequence with initial term u0 and common ratio q. In this
case we know that the expression of un is given by un = u0 · q n . Therefore
lim un = u0 · lim q n .
n→+∞ n→+∞
We can reformulate his theorem in this way: if (un ) is a geometric sequence with
common ration q, then,
1. when |q| < 1, we have
lim un = 0
n→+∞
lim vn = 0.
n→+∞
We can define sequences which have similar properties to both arithmetic and
geometric sequences. They are called arithmetico-geometric sequences.
un+1 = 4 un + 3, ∀ n ≥ 0,
is an arithmetico-geometric sequence.
Remark 3.3.3. We point out that in defintion 3.3.1,
• when b = 0 and a ∈ R\{0 ; 1} the sequence (un ) is geometric.
f (x0 ) = x0 .
un+1 = a un + b, ∀n ≥ 0.
Theorem 3.3.7. Let a and b be two nonzero real numbers such that a 6= 1. We
consider the sequence (un ) defined by
u0 ∈ R
un+1 = a un + b, ∀ n ≥ 0.
un+1 = 2 un + 3, ∀ n ≥ 0.
We define the function f (x) = 2 x + 3. The fixed point of this function is given by
x = −3. Then, the sequence
vn := un + 3
is geometric with initial term v0 = u0 + 3 = 5 and common ratio q = 2.
Example 3.3.9. Let (un ) be the sequence given by
u0 = −2
un+1 = −2un + 1, ∀ n ≥ 0.
un+1 = a un + b, ∀ n ≥ 0,
for some real number a 6= 1. Then the explicit form of (un ) is defined by (3.3.1).
un+1 = 2 un + 3, ∀ n ≥ 0.
Therefore we have
3 n 3
un = 2 − 2 + = 5 · 2n − 3
1−2 1−2
3.3. ARITHMETICO-GEOMETRIC SEQUENCES 89
un+1 = 3 un + 2, ∀ n ≥ 0.
Hence we have 2 n
un = 1 − 3 − 1 = 2 · 3n − 1
1−3
Exercise 3.3.13. Determine the explicit form of the following sequences
u0 = −7 u0 = 4
1. 2.
un+1 = −5 un + 2, ∀ n ≥ 0, un+1 = 8 un − 6, ∀ n ≥ 0.
b
un = ·
1−a
b
lim un = ·
n→∞ 1−a
b
Now, we suppose that u0 6= 1−a and for every n ≥ 0 we define the geometric
b
n
sequence u0 − 1−a · a . Hence, theorem 3.2.23 implies that
1. if |a| < 1
b n
lim u0 − ·a =0
n→+∞ 1−a
2. if |a| > 1
b n
lim u0 − ·a = ±∞.
n→+∞ 1−a
un+1 = a un + b, ∀ n ≥ 0.
Proof. Let (un ) be as theorem 3.3.14. Using (3.3.1) we obtain the following expres-
sion b n b
un = u0 − a + ·
1−a 1−a
Using the discussion above we obtain the teorem .
lim un = ±∞.
n→+∞
b 1 1 3
Since u0 − =1− = 1 + = , we have
1−a 1−3 2 2
lim un = +∞.
n→+∞
Exercise 3.4.1. We consider the sequence (un ) defined by u0 = 1 and for any
natural number n ≥ 0
un+1 = 2 un + 12.
For any natural number n we define the sequence vn = un + 12.
1. Establish that vn is a geometric sequence.
2. Define the explicit expression of the term vn for any natural number n ≥ 0.
3. Deduce from this the expression of the term un for every n ≥ 0.
4. Investigate the limit of (un ).
vn = 13 · 2n , ∀ n ≥ 0.
un = vn − 12 = 13 · 2n − 12.
lim 13 · 2n = +∞.
n→+∞
Hence we have
lim un = +∞.
n→+∞
The sequence un diverges.
Exercise 3.4.2. In Sahelian countries the lakes are facing drought problems.
The ” lac Rose” in Senegal is not an exception to this rule. Indeed, each year
the ” lac Rose ” loses 2% of its surface. We assume that the surface of this lake
3, 000, 000 m2 in 2021.
3. Specify the nature of the sequence un .
4. Define the explicit form of the term un for any natural number n.
5. Specify the year from which this lake can disappear if no backup policy is
taken.
Comments. Let u be any quantity and a a real number, such that, 0 ≤ a ≤ 100.
• If u increases
of a%, then, to find the new value of u we have to multiply u
a
by 1 + 100 ·
• If u decreases
of a%, then, to find the new value of u we have to multiply u
a
by 1 − 100 ·
• Here we consider that the year 2021 corresponds to the year 0 of the study.
Then, the surface in 2021 is denoted u0 .
• Hence in 2022, we have u1 = 1 − 100 2
u0 = 0.98 u0 ·
• In 2023, (the year 2 of the study), we have u2 = 0.98 u1
• If un is the surface of the lake in 2021 + n, the surface in 2021 + n + 1 is given
by un+1 . Then, we have
un+1 = 0.98 un .
• If (un ) is a geometric sequence, with initial term u0 and common ratio q, then
un = u0 · q n .
we can imagine that for n greater enough the lake will disappear.
Now we assume that the lake disappear when its surface is less than 100 m2 .
Hence, one has
n n 100 1
3 · 106 (0.98) ≤ 100 ⇐⇒ (0.98) ≤ =
3 · 106 3 · 104
− ln(3) − 4 ln(10)
n ln (0.98) ≤ − ln 3 · 104 = − ln(3) − 4 ln(10) =⇒ n ≥
·
ln(0.98)
Consequently, we have n = 511. We conclude that, if any preservation policy is not
taken, the lake will disappear in 2532.
3.4. EXERCISE SESSION 93
u2 = 186 + 36 = 222.
2. Let (un ) be the price to pay for the year 2021 + n. Then, in 2021 + n + 1 she
has to pay un+1 . In this case we obtain
un+1 = un + 36.
3. As we can observe it, the sequence (un ) is arithmetic with initial term u0 = 150
and common difference d = 36. Hence for any natural number n, we have,
un = u0 + n d = 150 + 36 n.
un+1 = 1 un ,
∀n≥0
4
1. Precise the nature of the sequence (un ).
2. Define the explicit form the term un , for all n ≥ 0.
3. Find the limit of (un ).
un+1 = q un .
1 − q n+1
sn = u0 · ·
1−q
Solution. 1. The sequence (un ) is geometric wih initial term u0 = 2 and common
ratio q = 41 ·
2. Let n ∈ N one has n
1
un = u0 · q n = 2 · ·
4
1
3. Since q = 4 < 1 we deduce that
n
1
lim un = lim 2 · = 0·
n→+∞ n→+∞ 4
Exercise 3.4.5. Let (un ) be the arithmetic sequence with initial term u0 = 3 and
common difference 7.
1. Define the recursive and the explicit forms of the sequence (un ).
un+1 = un + d, ∀ n≥0
un = u0 + n d = a + n d.
Solution. Now, we consider the arithmetic sequence (un ) with initial term u0 = 3
and common difference d = 7.
1. The recursive form of (un ) is given by the following formulas
u0 = 3,
un+1 = un + 7, ∀ n ≥ 0.
un+1 − un = 7 > 0.
3.5 Problems
Problem 3.5.1. In Senegal the climate change is of a major concern. That is
why, populations are taking some local initiatives. Young peoples of the association
”Ndiarignou Garab” (the usefulness of trees in wolof ) decide to take care of the
forest which is near to their village. This forest is composed out of 50000 big trees
in 2010. Their strategy consist of cutting 5% of the old trees and replant 3000 young
trees each year.
We denote by un the number of thousands of big trees which are in the forest in
the ongoing year 2010 + n. We agree that in 2010, we have u0 = 50.
1. Compute u1 , u2 and define the expression of un for any natural number n.
2. For every natural number n we define the sequence (vn ), in this way
vn = un − 60.
a. Establish that (vn ) is a geometric sequence (Hint: specify its common ration
and its initial term).
b. Determine the explicit expression of vn for any natural number n.
3. Deduce from 2. the explicit expression of un for any n ∈ N
4. Study the convergence of (un ).
5. Can the number of the trees of the forest exceed 60000 ?
Problem 3.5.2. We consider the sequence Pn defined by
P0 = 4
Pn+1 = − 2 Pn + 5,
∀ n ≥ 0.
3
1. Compute P1 and P2 .
2. We define the sequence Qn by Qn = Pn − 3 for every n ∈ N.
3.5. PROBLEMS 97
a. Prove that Qn is a geometric sequence
b. Determine the explicit form of Qn for every n ∈ N.
3. Deduce from 2.b. the expression of Pn for any natural number n.
4. Determine the limit of Pn .
Problem 3.5.3. Let (vn ) be the sequence given by v0 = 3 and for every
1
vn+1 = vn .
2
1. Determine the explicit form of (vn ).
2. Study the convergence of the sequence (vn ).
Problem 3.5.4. Let (an ) and (bn ) be two sequences, such that, a0 = 0, b0 = 12
and for every n ≥ 0,
2 an + bn
an+1 =
3
a + 3 bn
bn+1 = n
·
4
1. For every natural number n we define the sequence (un ) by
un = bn − an .
vn = 3an + 4bn .
23
vn = u n −
18
and prove that (vn ) is geometric.)
98 CHAPTER 3. ARITHMETIC AND GEOMETRIC SEQUENCES
23
2. Show that for any n ∈ N we have un ≥ 18 ·
2
Problem 3.5.6. Let (un ) be the sequence which satisfies u1 = 5 and for every
n ≥ 1,
1 2
un+1 = un + ·
5 5
1. Prove that (un ) is bounded above by 12 ·
We consider the arithmetic sequence (vn ) with initial term v0 = 16 and common
difference 8.
1. Justify that
n−1
X
sn = vk = 4n2 + 12n.
k=0
Problem 3.5.9. We consider the sequence (un ) defined by u0 = 1 and for all n ≥ 0,
1
un+1 = un + 4.
3
For any n ≥ 0 we set vn = un − 6.
1. Prove that (vn ) is a geometric sequence.
2. For any natural number n define the explicit form of vn .
3. Deduce from 2 that n
1
un = −5 + 6.
3
a. Prove that wn+1 = wn + 2 and define the explicit form of the term wn for
all natural number n.
3. Prove that
2n − 1
un = ·
2n
Pn
4. For any natural number n, we set sn = k=0 uk . Prove that
2n + 3
sn = 2 · ·
2n
un+1 = 4un − 1 ,
∀ n ≥ 0.
un + 2
1. Establish that for every n ≥ 0, un − 1 > 0.
2. For every natural number n, we define the sequence (vn ) by
1
vn = ·
un − 1
100 CHAPTER 3. ARITHMETIC AND GEOMETRIC SEQUENCES
un+1 = 0.98 un + 2, ∀n ≥ 0.
Contents
4.1 Generalities on Vectors . . . . . . . . . . . . . . . . . . . 102
4.2 Elementary Operations on Vectors . . . . . . . . . . . . 104
4.2.1 Sum of Vectors . . . . . . . . . . . . . . . . . . . . . . . 104
4.2.2 Multiplication by a Real Number . . . . . . . . . . . . . 104
4.3 Basis in the Plane . . . . . . . . . . . . . . . . . . . . . . 105
4.4 Scalar Product in the Plane . . . . . . . . . . . . . . . 109
4.4.1 Angle Between Two Vectors . . . . . . . . . . . . . . . . 110
4.4.2 Scalar Product of Two Vectors . . . . . . . . . . . . . . 110
4.5 Transformations in the Plane . . . . . . . . . . . . . . . 114
4.6 Determinant of 2 × 2 and 3 × 3 Matrices . . . . . . . . . 115
4.7 Introduction to Complex Plane . . . . . . . . . . . . . . 117
4.8 Lines in The Plane . . . . . . . . . . . . . . . . . . . . . . 119
4.9 Exercise Session . . . . . . . . . . . . . . . . . . . . . . . 122
4.10 Problems . . . . . . . . . . . . . . . . . . . . . . . . . . . 127
This chapter deals with geometry in the plane. We started with the plane
because, we thought that, it is more familiar to students. Another pedagogical
reason is that in the plane the represention of transformations are simplest and
most of the time we can draw them. The study of the geometry of the plane will
be done through vectors. To define a vector we need to know its direction. Here,
we mean by direction a set of parallel lines. For instance we have the horizontal
direction, the vertical direction or the oblique direction.
We start by giving the general definition of a vector. As we did it for complex
numbers, we will define elementary operations on the set of vectors. We define sums
of vectors and multiplication of a vector by a real number. In addition of these two
operations, we will introduce the concept of collinearity.
101
102 CHAPTER 4. INTRODUCTION TO GEOMETRY IN THE PLANE
Definition 4.1.1. The direction of a line (D) is the measure of the angle θ that
it makes with any horizontal line.
Remark 4.1.2. Let (D) be a line in the plane. We denote by θ the angle between
this line and a horizontal line. Then,
• When the angle θ is zero or π, we say that the line is horizontal
• when θ is equal to π/2 or −π/2, the line is called a vertical line.
Now, we are in a position to define rigorously what we mean by a vector.
• direction,
• orientation (or sense),
• norm (length or magnitude).
The orientation is the sense of displacement. The norm is the magnitude or the
length of the vector.
Notation: A vector ~u can be represented geometrically by an arrow in the plane
as follows
→
−
u
4.1. GENERALITIES ON VECTORS 103
The original point of the arrow is call the origin of the vector. The point at the tip
of the arrow is the end point of the vector. For instance, the vector with origin A
−−→
and end point B is denoted AB. It is represented as below
B
A −−→
AB
−−→ −−→
Notation: The norm of the vector AB is denoted AB .
From definition 4.1.3 we deduce that:
Lemma 4.1.4. Two vectors ~u and ~v are equal if they have the same direction,
the same sense and the same norm. In this case we write ~u = ~v
→
−
v
→
−
u
Definition 4.1.5. The vector ~u, such that, the origin and the end point coincide
→
−
is called the null or the zero vector. We denote it 0 .
We point out that, the zero vector does not have any specific direction and its
norm is 0. If ~u is any vector in the plane , the null vector satisfies
→
− →
− →
− −
u + 0 = 0 +→ u =→ −u.
Now take two vectors ~u and ~v in the plane such that ~u + ~v = ~0. This identity is
equivalent to ~v = −~u.
As we can see it, the vectors ~u and −~u have same direction and same norm but
opposite senses.
Lemma 4.1.6. For any vector ~u in the plane there exists a vector ~v in the plane
such that
~u + ~v = ~0.
The vector ~v is called the opposite vector of ~u. It is denoted −~u.
This lemma implies the following definition:
Definition 4.1.7. Two vectors →
−u and →
−v are said to be opposite if they have the
same direction, the same norm and opposite sense (orientation). In this case we
write ~v = −~u
→
−
v
→
−
u
104 CHAPTER 4. INTRODUCTION TO GEOMETRY IN THE PLANE
Definition 4.1.8. Any vector ~u which has norm 1, is called a unit vector.
In the next section we will show how to perform addition of vectors. We define
also the vector obtain by multiplying a ~u by a real number k.
Definition 4.2.1. The sum of two vectors ~u and ~v denoted ~u +~v is the vector with
origin the origin point of →
−
u and end point the end point of → −
v . It is represented
in this way
→
−
v
→
−
u
→
−
u +→
−
v
→
−
v1
→
− + →
−
v2 u
From this definition, we deduce that 0 · ~u is the vector with norm 0. This means
that 0 · ~u = ~0. The vector 1 · ~u = ~u is a vector wich has the same norm, the same
direction and same sense as ~u. This involves 1 · ~u = ~u. In the same way we have
k (`~u) = k · ` · ~u
→
−
v
→
−
u
2. (k + `) ~u = k ~u + ` ~u.
At this level one may ask how to check whether two vectors are collinear. The
first thing on which we will think is to use a ruler in order to verify if the vectors are
parallel. In the next section we will show that there is an easiest way to establish
that two vectors are collinear. The goal of the next section is to allows us to do
geometry without using compass and rulers. To be able to do this, we will associate
to any point and vector in the plane an ordered couple of real numbers. In this case
we say that we define coordinates. We will think in terms of coordinates instead of
vectors. To define coordinates we need to specify a frame.
Definition 4.3.1. Any couple of non-collinear vectors ~e1 and ~e2 define a basis
of the plane.
Theorem 4.3.2. Let (~e1 , ~e2 ) be a basis of the plane. We consider the vector →
−
v
in the plane. Then, there exist two real numbers x and y, such that,
→
−
v = x ~e1 + y ~e2 . (4.3.1)
The real numbers x and y are called the coordinates of the vector ~v in the basis
(~e1 , ~e2 ) and we write
x
~v (x , y) or ~u .
y
To perform calculations on vectors it is better to use coordinates.
Lemma 4.3.3. Let ~u (x , y) and ~v (x0 , y 0 ) be two vectors in the plane and k a
real number.Then,
1. the coordinates of the vector ~u ± ~v are (x ± x0 , y ± y 0 )
2. the coordinates of the vector k · ~u are (k x, k y).
From now on, we suppose that the point O is the origin of a frame and has
coordinates (0 , 0). Now we equip the plane with the orthonormal basis (~i , ~j). Let
−→
A be a point in the plane. We define the vector OA. Hence we can find two real
numbers xA and yA such that ,
−→
OA = xA~i + yA ~j.
−→
Thus, the real numers xA and yA are the coordinates of the vector OA. In this case,
we define the real numbers xA and yA as the coordinates of the point A. After this
update, we consider another point B in the plane with coordinates (xB , yB ), then,
−−→
we define the coordinates of the vector AB as follows
Definition 4.3.11. If A (xA , yA ) and B (xB , yB ) are two points in the plane,
−−→
the coordinates of the vector AB are defined by
−−→ xB − xA
AB (4.3.2)
yB − yA
Definition 4.3.14. If A (xA , yA ) and B (xB , yB ) are two points in the plane,
−−→
the norm of the vector AB is
−−→ p
AB = (xB − xA )2 + (yB − yA )2 . (4.3.3)
In other words two vectors are equal if they have the same coordinates.
Now we are in a position to use coordinates to characterize the collinearity of
vectors.
108 CHAPTER 4. INTRODUCTION TO GEOMETRY IN THE PLANE
Lemma 4.3.18. Let ~u (x , y) and ~v (x0 , y 0 ) be two vectors in the plane. Then,
the vectors ~u and ~v are collinear if and only if
x y 0 − x0 y = 0 (4.3.4)
Proof. Since ~u (x , y) and ~v (x0 , y 0 ) are collinear vectors, there exists some real
number k such that ~v = k~u. That is, (x0 , y 0 ) = k (x , y). This means that
x y0 = k x y and x0 y = k x y.
Therefore
x y 0 − x0 y = k x y − k x y = 0.
Conversely we assume x y 0 − x0 y = 0. Then, x 6= 0 and y 6= 0. Otherwise the
following cases can hold:
Case 1: x = 0 and y 6= 0. In this case one may have x0 = 0 or y 0 = 0.
• If we have x0 = 0 and y 0 6= 0 then the vectors ~u (0, y) and ~v (0, y 0 ) are
collinear.
• if x0 6= 0 and y 0 = 0 then y = 0. This involves ~u (0, 0) and ~v (x0 , 0) are
collinear.
Case 2: x 6= 0 and y = 0. Hence
Now we take the points A (xA , yA ),B (xB , yB ),C (xC , yC ) and D (xD , yD ). Using
the fact that two vectors are equal if they have the same coordinates. Then, we
obtain
4.4. SCALAR PRODUCT IN THE PLANE 109
xB − xA = xC − xD and yB − yA = yC − yD .
Exercise 4.3.24. Let A (3 , 0), B (0 , −6), C (−9 , −12) and D (−6 , −6) be four
points in the plane. Prove that ABCD is a parallelogram.
Lemma 4.3.25. If A (xA , yA ) and B (xB , yB ) are two points in the plane then,
the coordinates of the midpoint I of the segment line [AB] are given by
xA + xB yA + yB
xI = and yI = · (4.3.5)
2 2
0+5 5 1+2 3
xI = = and yI = = ·
2 2 2 2
Exercise 4.3.27. Find the coordinates of the midpoint of the segment line [AB]
1. A (3 , 5) and B (6 , 1),
2. A (0 , 7) and B (2 , 3),
One knows that a rectangle is a particular parallelogram which has four right
angles. This means that the consecutive sizes are arthogonal. There are several
methods to establish that two vectors are orthogonal. At our level the simplest way
consists of using the scalar product.
~v
~u
θ0
→
−
w1
→
−
w0
Remark 4.4.3. We point out that here we define the angle θ as the smallest coun-
terclockwise angle. The angle (~v , ~u) is equal to −θ.
The oriented angles share the following properties
Since we know the definition of an angle between two vectors, we are in a position
to define the scalar or the dot product of two vectors.
Definition 4.4.5. The scalar product or dot product of the vectors ~u and ~v is
the real number ~u · ~v defined as follows:
If we suppose the angle θ = (~u, ~v ) = 0, one has cos(θ) = 1. In this case the
following lemma holds:
~u · ~v = k~uk · k~v k.
~u · ~v = −k~uk · k~v k
π
If the angle (~u, ~v ) = 2, one has ~u · ~v = 0.
Proof. Let ~u and ~v be two orthogonal vectors in the plane. Therefore we have
cos (~u , ~v ) = cos ± π/2 = 0.
We conclude that
~u · ~v = k~uk · k~v k cos ± π/2 = 0.
Conversly we consider two nonzero vectors ~u and ~v , such that, ~u · ~v = 0. As we
supposed that ~u and ~v are two nonzero vectors, we have k~uk =
6 0 and k~v k =
6 0. This
yields cos (~u , ~v ) = 0. Hence the angle (~u , ~v ) = ±π/2. The vectors ~u and ~v are
orthogonal.
This means that the scalar product is commutative. In the same way we can prove
the distributivity of the scalar product with respect to addition of vectors. In other
words we have
~u · (~v + w)
~ = ~u · ~v + ~u · w.
~
Let ~u be a vector in the plane, then the angle (~u , ~u) = 0. Hence, ~u · ~u = k~uk2 .
This involves
Proposition 4.4.10. If ~u and ~v are two vectors in the plane and k a real number
then,
~u · (k~v ) = (k ~u) · ~v = k (~u · ~v ).
Definition 4.4.11. The scalar product of the vectors ~u (u1 , u2 ) and ~v (v1 , v2 ) is
defined by
→
−u ·→−
v = u1 v1 + u2 v2 . (4.4.3)
Example 4.4.12. We consider the vectors ~u = (2, −6) and ~v = (5, 2). Then,
~u · ~v = 2 · 5 + (−6) · 2 = 10 − 12 = −2.
Theorem 4.4.15. If ~u and ~v are two nonzero vectors such that θ = (~u, ~v ), then
~u · ~v
cos(θ) = · (4.4.4)
~u ~v
Example 4.4.16. Find the angle between the vectors ~u = (1 , −2) and ~v = (−3 , 6).
Using (4.4.4) we have
Using the scalar product, we can also prove the following theorem
Theorem 4.4.19. Let ~u and ~v be two vectors in the plane. Then, ~u and ~v are
orthogonal if and only if
~u + ~v = ~u − ~v . (4.4.6)
When ~u and ~v are two vectors which are not necessarily orthogonal, one can
establish this theorem.
A A’
→
−
u
Definition 4.5.3. Let A, A0 and I be three points in the plane. We say that A0
is the image of A in the homothety with center I and ratio k 6= 0, if
−→0 −→
IA = k IA.
We denote A0 = hI , k (A).
−→0
A IA A’
I −→
IA
We denote
A0 = RI,θ (A).
Theorem 4.5.6. Let A, A0 and I be three points in the plane and θ a real number.
We assume
A0 = RI , θ (A).
Then, A and A0 belong to the circle with center I and radius R = IA.
Another way to check whether two vectors are collinear in the plane is to con-
struct a matrix with these two vectors and to compute the determinant of this
matrix. If the determinant of A is 0, then the two vectors are collinear (linearly
dependant). Otherwise they are not collinear (linearly independant). In the next
section we introduce determinant of matrices
The matrix C is called the null or zero matrix and the matrix E is the identity
matrix .
a b
Definition 4.6.3. The determinant of a 2 × 2 matrix A = is defined by
c d
a b
det(A) = = ad − bc. (4.6.1)
c d
116 CHAPTER 4. INTRODUCTION TO GEOMETRY IN THE PLANE
We point out that in this course we will only consider determinants of square
matrices.
3 −2
Example 4.6.4. We consider the matrix A = . Then,
4 1
3 −2
det(A) = = 3 · 1 − (−2) · 4 = 3 − (−8) = 11
4 1
a1 a2 a3
b b3 b1 b3 b b2
b1 b2 b3 = a1 2 − a2 + a3 1 (4.6.2)
c2 c3 c1 c3 c1 c2
c1 c2 c3
1 −2 0
2 −4 2 −4 2 2
2 2 −4 = 1 − (−2) +0
−4 −1 3 −1 3 −4
3 −4 −1
= 1 · (−18) + 2 · 10 + 0 · (−14) = 2.
Theorem 4.6.11. Let A be a matrix such that, two rows are collinear. Then,
det(A) = 0.
Theorem 4.6.12. Let A be a matrix such that two columns are collinear.Then,
det(A) = 0.
Proposition 4.7.4. Let ~u (z~u ) and ~v (z~v ) be two vectors in the complex plane.
Then,
z~v
(~u, ~v ) = arg = arg(z~v ) − arg(z~u ).
z~u
√
Example 4.7.5. Let ~u ( 3 + i) and ~v (1 + i) be two vectors in the complex plane.
Then ,
√
~i , ~u = arg(z~u ) = arg 3 + i = π and ~i , ~v = arg 1 + i = − π ·
6 4
From this we deduce that
1+i π π 3π 2π π
~u , ~v = arg √ = − = − = ·
3+i 4 6 12 12 12
√ √
Exercise 4.7.6. We consider the vectors ~u (−2+2 i), ~v (1+i 3) and → −w (−1+i 3).
Determine the measure of the following angles (~u, ~v ), (~u , →
−
w ), (→
−
w , ~v ).
In section 4.5 we defined some transformations. Since we know that the complex
numbers and the plane are the same, we redefine these transformations in terms of
complex numbers. In other words we study these transformations by using their
affixes.
Lemma 4.7.7. Let ~u (z~u ) be a vector in the plane. We consider two points
M (zM ) and M 0 (zM 0 ), such that, M 0 = τ~u (M ). Then,
z~u = zM 0 − zM .
This lemma allows to express the affix of M 0 as a function of the affix of the
vector ~u and the point M. In other words we have
zM 0 = z~u + zM .
Lemma 4.7.8. Let A (zA ), B (zB ) , C (zC ) and D (zD ) be four points in the
complex plane. Then, ABCD is a parallelogram if and only if
zB − zA = zC − zD .
Lemma 4.7.9. If I (zI ), M (zM ) and M 0 (zM 0 are three points in the plane and
k ∈ R such that, M 0 = hI , k (M ), then,
zM 0 = 1 − k zI + k zM . (4.7.1)
Lemma 4.7.10. Let M (z), M1 (z1 ), M2 (z2 ) and O (0) be four points points in
the complex plane. We consider two real numbers k and θ. Then,
3. when M2 = Rθ (M1 ), z2 = ei θ z1
Most of the readers can be confused. Because we are doing geometry in the
plane without saying anything on lines. We preferred to start by explaining points
and vectors. To reassure readers we introduce the next section which is devoted to
lines.
yB = a xB + b,
where the unknowns are a and b. In this case , if xB 6= xA , one can establish that
yB − yA
a= ·
xB − xA
Getting a we deduce that
yB − yA
b = yA − xA .
xB − xA
Example 4.8.1. Define an equation of the line that passes through the points
A (3, 2) and B (1, 4).
120 CHAPTER 4. INTRODUCTION TO GEOMETRY IN THE PLANE
Definition 4.8.3. A direction vector of the line (D), is any vector ~u which is
parallel to (D).
a x + b y + h = 0. (4.8.1)
2 x + 3 y + 4 = 0.
a b0 − a0 b = 0.
Example 4.8.9. We consider the lines (D) and (D0 ) with respective equations
2 x + 3 y + 1 = 0, 6 x + 9 y + 2 = 0.
4.8. LINES IN THE PLANE 121
Therefore we have
6 × 3 − 2 × 9 = 0,
Now we consider a line passing through a point A (xA , yA ) with direction vector
−−→
~u (a , b). Let M (x, y) be another point of this line. The vectors AM and ~u are
collinear. Therefore we have (x − xA ) b − a(y − yA ) = 0. This can be rewritten as a
determinant of the matrix
x − xA a
= 0.
y − yA b
Lemma 4.8.11. Let (D) be a line passing through A (xA , yA ) with direction
vector ~u (a , b). Then, for every point M (x , y) ∈ (D) we have
x − xA a
= 0. (4.8.2)
y − yA b
Let (D) be a line passing through A(xA , yA ) with direction vector ~u (a, b). We
−−→
consider the point M (x , y) in (D). We see that the vectors AM and ~u are collinear.
This means, there is a real number k such that, (x − xA , y − yA ) = k (a , b). This
is equivalent to the following system:
x = xA + k a
, (4.8.3)
y = yA + k b, k ∈ R.
Definition 4.8.12. A parametric equation of the line (D) which passes through
A (xA , yA ) with a direction vector ~u (a, b) is defined by
x = xA + k a
k ∈ R.
y = yA + k b,
122 CHAPTER 4. INTRODUCTION TO GEOMETRY IN THE PLANE
Example 4.8.13. Let (D) be the line that passes through B (2, 3) with direction
vector ~u (2, 6). A set of parametric equations of (D) is
x = 2 + 2k
y = 3 + 6 k, k∈R
Exercise 4.8.14. Define a set of parametric equations for each of the following
lines which passes through A with direction vector ~u
1. A (2 , 1) and ~u (3 , 5),
−→ 3 −→
IA = ID
4
4.9. EXERCISE SESSION 123
is equivalent to
− 27 − xI
3 3 − xI
= 5 .
2 − yI 4 2 − yI
This leads to the following system
7 9 3
− 2 − xI = − xI
4 4
2−y 15 3
I = − yI
8 4
From this we deduce that xI = −23 and yI = − 21 · We conclude that I − 23 ; − 12 .
zA = 1 + i, zB = −1 + i, zC = −1 − i and zD = 1 − i.
Let R be the rotation of center C and angle −π/3. We denote E = R(B) and
F = R(D).
1. Define the expression of the rotation R.
√
2. a. Prove that zE = −1 + 3.
b. Determine zF .
zA − zE
c. Establish that
zA − zF
−→ −→
d. Deduce from c. that the vectors AE and AF are collinear.
Solution. 1. Let M (z) be a point in the plane. We consider its image M 0 (z 0 ) in
the rotation with center C(zC = −1 − i) and angle −π/3. Then, we have
π
z 0 − zC = exp −i (z − zC ).
3
This leads to the following formula
π
z 0 = −1 − i + exp −i (z + 1 + i). (4.9.1)
3
2.
a. From (4.9.1) we that
π π
zE = −1 − i + exp −i (−1 + i + 1 + i) = −1 − i + 2i exp −i
√ 3 3
= −1 + 3.
b. Since F is the image of D in the rotation with center C and angle −π/3, we have
π π
zF = −1 − i + exp −i (1 − i + 1 + i) = −1 − i + 2 exp −i
√ 3 3
= (−1 − 3)i.
4.9. EXERCISE SESSION 125
d. From c we can see that there exists a real number k such that
zA − zE = k(zA − zF ).
Step 3 Our goal now is to prove that the property holds for k + 1. We want to
establish that k+1
1 3(k + 1)π
zk+1 = exp i .
2 4
Using (4.9.2) we have √
2
zk+1 = − 1 + i zk .
4
One deduces from Step 2 that
√ k
2 1 3π 1 3kπ
zk+1 = (−1 + i)zk = exp i · exp i
4 2 4 2 4
k+1
1 3(k + 1)π
= exp i .
2 4
• We point out that the property holds also for 0. Hence we can start our
induction with 0 instead of 1.
Exercise 4.9.5. Let A, B, S and Ω be four points in the plane that are respectively
associated to the complex numbers a = −2 + 4i, b = −4 + 2i, s = −5 + 5i and
ω = −2 + 2i. We define the points C and D, such that
−→ −→ −→ −→
SC = 3SA and SD = 3SB.
This gives
zC = −6 + 12i + 10 − 10i = 4 + 2i.
4.10. PROBLEMS 127
−→ −→
In the same way we can establish that the identity SD = 3SB is equivalent to
One obtains
zD = 3zB − 2zS = −12 + 6i + 10 − 10i = −2 − 4i
2. A look on the affix of the points A, B, C and D allows us to emphasize that
√
OA = OB = OC = OD = 20.
√
Therefore, these points are on the circle with center O and radius R = 20.
4.10 Problems
Problem 4.10.1. Prove that for all points A, B, C and D in the plane we have
−→ −−→ −→ −−→
OA − OB + AC = BC.
→
−
a. →
−
a 1 , 2 and b 2, 4
→
−
b. →
−
a 1 , 2 and b − 2, 1
Problem 4.10.8. Let (d) be the line with direction vector ~u (1 , 2) that passes
through A (1 , 5). Define a set ofparametric equations for the line (d).
Problem 4.10.9. Let ABCD be a parallelogram. We consider the points I and E
such that
−−→ −
→ −→ 1 −→
AB = 2 AI and IE = ID.
3
−→ −−→ −−→
1. Find α and β in R, such that, AC = α AB + β AD.
2. Prove that
−→ 2 −→ 1 −−→
AE = AI + AB.
3 3
−→ −−→ −−→
3. Deduce from this, there exist α1 and α2 in R, such that, AE = α1 AB+α2 AD.
4. Prove that A, E and C belong to the same line.
Problem 4.10.10. Let M be a point of the complex plane with affix z. Find the
set of all points M such that
z−3
1. = 1,
z−5
√
z−3 2
2. = ·
z−5 2
Problem 4.10.11. Let M be a point of the complex plane with affix z. Find the
set of all points M such that
1. |z| = 3,
2. |z − i| = 5,
3. |z − 2 i| = |z + 2 − 3 i|.
Problem 4.10.12. Let A (−2 − 2 i), B (2), C (2 + 4 i), D (−2 + 2 i), E (4 + 6 i) and
F (−2 i) be six points of the complex plane
1. Show that ABCD is a parallelogram.
2. Prove that the triangle CDE is isosceles.
Problem 4.10.13. Let A, B, C and D be four points in the plane such that
2. Determine an argument of Z.
4.10. PROBLEMS 129
2. Show that A, B, C and D belong to a same circle. Give the radius of this
circle.
−→ −−→
3. Let z1 be the affix of the vector AC, z2 be the affix of the vector BD, z3 be
−−→ −−→
the affix of the vector AB and z4 be the affix of DC.
√
a. Prove that z2 = z1 3.
b. Calculate |z3 | and |z4 |.
c. Prove that ABCD is a trapezoid.
z = z−2 .
b. Deduce from this the points B 0 and C 0 which are associated to B and C.
c. We define G as the center of gravity of the triangle OAB. Determine the
point G0 which is associated to G
Problem 4.10.17. Let A and B be two points which are associated to the complex
numbers zA = i and zB = exp −i 5π 6 .
Contents
5.1 Basis in Space . . . . . . . . . . . . . . . . . . . . . . . . 132
5.2 Scalar Product in Space . . . . . . . . . . . . . . . . . . 135
5.3 Lines in Space . . . . . . . . . . . . . . . . . . . . . . . . 137
5.4 Vectorial Product . . . . . . . . . . . . . . . . . . . . . . 140
5.5 Study of Planes in Space . . . . . . . . . . . . . . . . . 145
5.6 Some Properties of the Mixed Product . . . . . . . . . 150
5.7 Exercise Session . . . . . . . . . . . . . . . . . . . . . . . 152
5.8 Problems . . . . . . . . . . . . . . . . . . . . . . . . . . . 156
In the previous chapter we studied vectors in the plane and we pointed out that
to any vector ~u in the plane, we can associate two real numbers x and y. The real
numbers x and y are called the coordinates of the vector ~u. The aim of this chapter
is to continue the study of vectors in 3-dimensional space. The idea is at first to
extend the notion of coordinates, scalar product, orthogonality, collinearity,· · · · We
will also define new operations : the vector or cross product and mixed product.
We remind that to locate a point or a vector in space we associate to this
vector or point three real numbers x, y and z. These real numbers represent their
coordinates. We denote ~u (x , y , z) for the vector ~u. When it comes to a point A
we write A (xA , yA , zA ).
These coordinates will allow us to simplify computations. They will give us the
possibilty to compute easily area and volume of domains.
We make the following update. Throughout this chapter all definitions which
do not use coordinates are the same in plane as well as in 3-dimensional space.
This chapter is organized in this way. In section 1 we explain how to get coordi-
nates by using a basis. In section 2 we define the scalar product in three dimension.
Using the scalar product we will characterize lines in space. This will be done in
131
132 CHAPTER 5. INTRODUCTION TO GEOMETRY IN SPACE
section 3. The section 4 is devoted to the cross product. Using the cross product we
will study planes in section 5. To end this chapter we introduce the mixed product.
Mixted product whose application will allow to determine volumes of some figures.
w
~ = α ~u + β ~v . (5.1.1)
In other words any three vectors which are not in the same plane define a basis
of the space.
Definition 5.1.4. We say that the basis ~u, ~v , w
~ is orthogonal if
~u ⊥ ~v , ~u ⊥ w
~ and ~v ⊥ w.
~
Definition 5.1.5. The basis ~u, ~v , w
~ is orthonormal if it is orthogonal and the
vectors ~u, ~v and w
~ satisfy
thonormal basis.
~ of the space. To any vector →
Now we fix a basis (~u, ~v , w) −
χ in the space we
associate three real numbers x, y and z such that
→
−
χ = x ~u + y ~v + z w.
~ (5.1.2)
Definition 5.1.7. The real numbers x, y and z in (5.1.2) are called the coordi-
nates of →
−
χ in the basis (~u, ~v , w).
~
One may think that only orthonormal bases allow to define coordinates. Here
we draw readers attention on the fact that we can define coordinates in any type of
basis. However the use of orthonormal bases make the calculations simplest.
To make the presentation clear we mostly consider orthonormal bases. The
reason of this is Lemma 5.2.8, in section 2. This lemma claims that we can always
transform any basis to an orthonormal basis. This process is called Gram-Schmidt
process.
Definition 5.1.8. Any vector ~u in space with norm k~uk = 1 is called a unit
vector.
Remark 5.1.9. We can define unit vectors in any direction, not only in the direc-
tion of the axis. A unit vector which has the same direction as the nonzero vector
~u can be defined by
~u
·
k~uk
Example 5.1.10. Le ~v (1 , 1 , 0) be a vector in space. Then, the vector
1 1
~e~v √ , √ , 0
2 2
is a unit vector which has same direction as ~v .
Example 5.1.11. We consider the vector ~u (1 , −1 , 2). We define vector
√ !
1 −1 2
~eu √ , √ , √ .
6 6 3
The vector ~eu is a unit vector which has same direction as ~u.
From now on, we fix the following notations. The unit vectors along the x-axis,
y-axis and z-axis are respectively denoted ~i (1, 0, 0), ~j (0, 1, 0), and ~k (0, 0, 1).
It follows immediately that each vector in space can be expressed uniquely in
terms of vectors ~i, ~j, and ~k.
Now we take a vector ~u x, y, z . The vector ~u can be written in this way
~u = x~i + y ~j + z ~k.
134 CHAPTER 5. INTRODUCTION TO GEOMETRY IN SPACE
Exercise 5.1.13. Write the following vectors as combinations of the vectors ~i, ~j
and ~k
1. ~a 0, 2, 1 ,
2. ~b 2, 6, 0 ,
3. ~u 4, 0, 5 ,
4. ~v (−2 , 1 , 0),
~ (−7 , −1 , 5).
5. w
nates of the point O as (0, 0, 0). Now we take another point A in the space. Since
−→
OA is a vector there are three real numbers xA , yA and zA , such that,
−→
OA = xA ~i + yA ~j + zA ~k.
The real numbers xA , yA and zA are called the coordinates of point A and we denote
A xA , y A , z A .
Definition 5.1.14. If A xA , yA , zA and B xB , yB , zB are two points in the
−−→
space, the coordinates of the vector AB are
xB − xA
−−→
AB yB − yA
zB − zA
Definition 5.1.15. If A xA , yA , zA and B xB , yB , zB are two points in the
−−→
space, the distance between A and B or the norm of the vector AB is
−−→ p
AB = (xB − xA )2 + (yB − yA )2 + (zB − zA )2 .
Example 5.1.16. Take the points A 1, 3, 2 and B 5 , 0, 4 . Then, we have
−−→ p √ √
AB = (5 − 1)2 + (0 − 3)2 + (4 − 2)2 = 16 + 9 + 4 = 29.
Exercise 5.1.17. We consider the points A 1, 2, 0 , B 3, 2, 3 , C 0, 0, 1 and
−−→ −→ −−→ −−→ −−→ −−→
D 5, 6, 9 . Find the norm of the vectors AB, AC, AD, BC, BD and DC.
One may wonder if Pythagor’s Theorem remained valid. To convince readers
that Pythagore’s Theorem holds, we state the following theorem which only use
definition of a norms.
5.2. SCALAR PRODUCT IN SPACE 135
Theorem 5.1.18. Let A, B and C be three points in space such that the triangle
ABC is right with hypotenuse [BC]. Then,
−−→ 2 −−→ 2 −→ 2
BC = AB + AC .
k~uk2 = ~u · ~u.
Lemma 5.2.3. Two nonzero vectors ~u and ~v are orthogonal if and only if
~u · ~v = 0.
Proof. Let ~u and ~v be two orthogonal vectors. Then the angle ~u , ~v = ±π/2. This
implies that cos(~u, ~v ) = 0. Therefore, we obtain ~u · ~v = 0.
Conversely we assume ~u and ~v are nonzero vectors such that, ~u · ~v = 0. By
definition ~u ·~v = k~uk k~v k cos(~u, ~v ) = 0. Since ~u and ~v are nonzero vectors, we obtain
cos ~u , ~v = 0. We conclude that ~u ⊥ ~v .
For vectors which are not orthogonal, on can prove the following general identity.
It is called the parallelogram identity.
Theorem 5.2.5. For any vectors ~u and ~v , we have
Proof. Let ~u and ~v be two vectors which are not necessarly orthogonal. Then, we
have
(∗) k~u − ~v k2 = k~uk2 − 2 ~u · ~v + k~v k2 and (∗∗) k~u + ~v k2 = k~uk2 + 2 ~u · ~v + k~v k2 .
Summing (∗) and (∗∗) we obtain (5.2.1).
As we know that any vector is defined by its coordinates, we should have a
definition of the dot product in terms of coordinates.
Definition 5.2.6. The scalar product of ~u u1 , u2 , u3 and ~v v1 , v2 , v3 is
~u · ~v = u1 v1 + u2 v2 + u3 v3 .
Lemma 5.2.7. If ~u u1 , u2 , u3 is a vector in space, then,
In the previous section we have said that we will always consider orthonormal
bases. This choce is justified by the following lemma:
Lemma 5.2.8. Any basis (~u, ~v , w) ~ of the space can be transformed to an or-
thonormal basis (~e1 , ~e2 , ~e3 ).
Proof. Let (~u, ~v , w)
~ be a basis of the space. We assume that there are three vectors
~e1 , ~e2 and ~e3 wich satisfy the following properties:
1. (~e1 , ~e2 , ~e3 ) is an orthonormal basis
2. the vectorial line defined by ~u is the same as the vectorial line defined by ~e1
3. the plane defined by (~u, ~v ) is the same as the plane defined by (~e1 , ~e2 )
4. The vectors (~u, ~v , w)
~ and (~e1 , ~e2 , ~e3 ) define the same space.
Now we have to construct the vectors ~e1 , ~e2 and ~e3 . We proceed as follows
Step 1: Construction of ~e1 . Using assumption 2 we can find λ ∈ R such that
~e1 = λ ~u.
This implies that k~e1 k = |λ| k~uk. Since k~e1 k = 1, we get
1
|λ| = ·
k~uk
In this case we define
~u
~e1 = ·
k~uk
5.3. LINES IN SPACE 137
Step 2: Construction of ~e2 . The hypothesis 3 allows to find two real numbers
α and β, such that, ~v = α ~e1 + β ~e2 . We set ~g2 := β ~e2 = ~v − α ~e1 . We have
0 = ~g2 · ~e1 = β~e2 · ~e1 = ~v · ~e1 − α~e1 · ~e1 .
This involves ~v · ~e1 − α = 0. Therefore α = ~v · ~e1 and ~g2 = ~v − ~v · ~e1 ~e1 . The
vector ~g2 is orthogonal to ~e1 , but it is not a unit vector. To get a unit vector
we set
~g2
~e2 = ·
k~g2 k
Step 3: Construction of ~e3 . To construct ~e3 , we use the assumption 4. There
are three real numbers a, b and c, such that,
w
~ = a ~e1 + b ~e2 + c ~e3 .
We define ~g3 := c ~e3 = w~ − a ~e1 − b ~e2 . Since (~e1 , ~e2 , ~e3 ) is an orthonormal basis
we have ~e3 · ~e1 = 0, ~e1 · ~e2 = 0 and ~e3 · ~e2 = 0. This yields a = w ~ · ~e1 and
b=w ~ · ~e2 . Hence
~g3 = w ~− w ~ · ~e1 ~e1 − w ~ · ~e2 ~e2 .
The vector ~g3 is orthogonal both to ~e1 and ~e2 . But ~g3 is not a unit vector.To
obtain a unit vector we consider
~g3
~e3 = ·
k~g3 k
Example 5.3.2. Below we represent the line (d) and a direction vector ~u
→
−
u (d)
138 CHAPTER 5. INTRODUCTION TO GEOMETRY IN SPACE
Let A (x0 , y0 , z0 ) be a point in space and (d) a line which passes through A with
direction the nonzero vector ~u (u1 , u2 , u3 ). We consider another point M (x , y , z)
−−→
on this line. The vectors ~u and AM are collinear. This means that we can find a
−−→
real number t such that , AM = t ~u. From the expressions of the vectors
−−→
AM = (x − x0 )~i + (y − y0 ) ~j + (z − z0 ) ~k and ~u = u1 ~i + u2 ~j + u3 ~k,
we deduce that
(x − x0 )~i + (y − y0 ) ~j + (z − z0 ) ~k = t u1 ~i + t u2 ~j + t u3 ~k.
−−→ −−→ −→
The relation AM = t ~u leads to the identity OM = OA + t ~u, that is equivalent to
−−→ −→
OM = OA + t u1 ~i + t u2 ~j + t u3 ~k. (5.3.1)
−−→ −→
Now we set ~r(t) := OM = x~i + y ~j + z ~k and ~r0 = OA = x0 ~i + y0 ~j + z0 ~k. Replacing
in (5.3.1), we obtain
~r(t) = ~r0 + t~u. (5.3.2)
−→ −−→
Using coordinates of vectors ~u, OA and OM , one can establish
The identity (5.3.3) is called vectorial equation of the line (d) which passe through
A (x0 , y0 , z0 ) with a direction vector ~u.
Definition 5.3.3. The vectorial equation of the line (d) that passes through the
point A (x0 , y0 , z0 ) with direction vector ~u (u1 , u2 , u3 ) is given by (5.3.3).
Example 5.3.4. The vectorial equation of the line (d) that passes through the point
A(−1, 2, 0) with direction vector ~v = 2~i − ~j + 3 ~k is:
Exercise 5.3.5. Define the vectorial equation of the line (d) passing through the
point A with direction vector ~u
1. A (2, 4, 5) and ~u (2, 4, 5),
2. A (1, 2, 0) and ~u (1, 0, 5),
3. A (5, 3, 2) and ~u (1, 4, 6).
−−→
Proposition 5.3.6. The vector AB is a direction vector of the line (AB)
Having a look on the identity (5.3.3), we can give the following interpretation
in terms of coordinates: x = x0 + t u1 , y = y0 + t u2 and z = z0 + t u3 . This leads
to this system
x = x0 + t u1 ,
y = y0 + t u2 , t∈R (5.3.4)
z = z0 + t u3 ,
5.3. LINES IN SPACE 139
Definition 5.3.7. A set of parametric equations for the line (D) that passes
through the point A (x0 , y0 , z0 ) with direction vector ~u (u1 , u2 , u3 ) is given by
(5.3.4)
Example 5.3.8. To define a set parametric equations for the line that passes
through A(1, 5, −3) and parallel to the vector ~v (2, −3, 3), we proceed in this way.
We use the definition of a set parametric equations, in order to obtain
x = 1 + 2t
y = 5 − 3 t, t ∈ R
z = −3 + 3 t
Example 5.3.9. Find a set of parametric equations for the line (AB) that passes
through the points A(1, 5, −3) and B(4, −1, 6).
To define a set of parametric equations, we need to find a direction vector to the
−−→ −−→
given line (AB). The vector AB provides a good choice: AB = (3, −6, 9). So the
equations are
x = 1 + 3k,
y = 5 − 6k, k ∈ R
z = −3 + 9k
Another set of parametric equations can be defined for the line (AB), if we
consider the point B instead of A
x = 4 + 3t
y = −1 − 6t, t ∈ R
z = 6 + 9t
−−→
Using any nonzero vector which is parallel to AB we can define other sets of
parametric equations. This allows to make the following remark:
Remark 5.3.10. In the space there are several possible parametrizations of a line.
In other words, a set of parametric equations for a given line are not unique.
Exercise 5.3.11. Define a set of parametric equations for the line that passes
through A and parallel to the vector ~u
x = x0 + t u1 , y = y0 + t u2 and z = z0 + t u3 .
140 CHAPTER 5. INTRODUCTION TO GEOMETRY IN SPACE
Hence the line (d) passes through the point A (x0 , y0 , z0 ). We fix the vector
~u (u1 , u2 , u3 ) as a direction vector of (d). Moreover we suppose that the real num-
bers u1 , u2 and u3 are non-zero real numbers. Then, one has
x − x0 y − y0 z − z0
t= = = · (5.3.5)
u1 u2 u3
Identity (5.3.5) is called the cartesian or symmetric equations of the line (d).
Definition 5.3.12. A cartesian equations of the line (d) that passes through the
point A (x0 , y0 , z0 ) with direction vector ~u is given by (5.3.5).
Example 5.3.13. Define a Cartesian equations for the line that passes through
A(3, −2, 2) and B(1, 3, −1)
One know that the vector
−−→
~u = AB = (1 − 3)~i + (3 − (−2))~j + (−1 − 2)~k = −2~i + 5~j − 3~k
is a direction vector of this line. Then, If we use the point A, we obtain these
symmetric equations
x−3 y+2 z−2
= = ·
−2 5 −3
Similarly, if we use the point B, we get another set of symetric equations
x−1 y−3 z+1
= = ·
−2 5 −3
Exercise 5.3.14. Define a cartesian equations of the line that passes through the
point A and parallel to ~u
Proposition 5.3.15. Two lines (d) and (d0 ) are parallel if they have the same
direction and two lines are orthogonal if their directions are orthogonal.
Definition 5.4.1. If ~u and ~v are two vectors in space we define the cross product
of ~u and ~v denoted ~u × ~v as the unique vector which satisfies
1. ~u × ~v is orthogonal both to the vectors ~u and ~v
2. ~u × ~v has norm
~u × ~v = k~uk k~v k sin(θ) . (5.4.1)
Notation: In the litterature we can find the following notation of the cross product
of ~u and ~v
~u ∧ ~v .
~u × ~v = 0.
~u × ~u = ~u × (−~u) = 0.
Proof. Indeed, if the vectors ~u and ~v are collinear, the angle θ = 0 + kπ, for some
k in Z. Therefore sin(θ) = 0. This implies ~u × ~v = 0.
Conversly we assume ~u × ~v = 0. Since ~u and ~v are nonzero vectors, we deduce
that sin(θ) = 0. This means that the vector ~u and ~v are collinear.
Definition 5.4.4. The cross product of the vectors ~u (u1 , u2 , u3 ) and ~v (v1 , v2 , v3 )
is
~i ~j ~k
u3 = u2 v3 −u3 v2 ~i− u1 v3 −u3 v1 ~j + u1 v2 −u2 v1 ~k. (5.4.2)
~u ×~v = u1 u2
v1 v2 v3
Let ~u and ~v be two vectors such that (~u , ~v ) = θ. Then, (~v , ~u) = −θ. Therefore
, sin(~v , ~u) = − sin(~u , ~v ). This implies that
~u × ~v = −~v × ~u.
142 CHAPTER 5. INTRODUCTION TO GEOMETRY IN SPACE
~u × (~v + w)
~ = (~u × ~v ) + (~u × w)
~ and (~u + ~v ) × w
~ = (~u × w)
~ + (~v × w).
~
Example 5.4.5. We consider the vectors ~u (1, 3 − 2) and ~v (2, 0, 1). Then, using
(5.4.2), we obtain
~i × ~j = ~k, ~j × ~k = ~i,
~k × ~i = ~j, ~j × ~i = −~k,
~k × ~j = −~i, ~i × ~k = −~j. (5.4.3)
We will just prove the first property. We fix ~i (1 , 0 , 0) and ~j (0 , 1 , 0), then
~i × ~j = (0 − 0) ~i − (0 − 0) ~j + (1 − 0) ~k = ~k.
~u × ~v = (u2 v3 − u3 v2 , u3 v1 − u1 v3 , u1 v2 − u2 v1 ).
Example 5.4.9. We consider the vectors that ~u (0, −1, 1) and ~v (1, 0, −2). Then,
one has
~i ~j ~k
−1 1 ~ 0 1 ~ 0 −1 ~
~u × ~v = 0 −1 1 = i− j+ k = 2~i + ~j + ~k.
0 −2 1 −2 1 0
1 0 −2
Example 5.4.10. To define a vector that is orthogonal both to vectors ~u (2, −1, 3)
and ~v (1, 4, −2), we consider the vector ~u × ~v . Consequently, we obtain
~i ~j ~k
−1 3 ~ 2 3 ~ 2 −1 ~
~u × ~v = 2 −1 3 = i− j+ k = −10 ~i + 7 ~j + 9 ~k.
4 −2 1 −2 1 4
1 4 −2
k~u × ~v k
sin(θ) = ·
k~uk k~v k
When the real number theta belongs to the interval [−π , 0[, we have to take
k~u × ~v k
sin(θ) = − ·
k~uk k~v k
~u × ~v
~u
~v
When ~u and ~v are nonzero vectors which are not parallel the direction of the
vector ~u × ~v can be determine as follows: We use the fingers of the right hand so
that the index finger represent the vector ~u, the middle finger represent the vector
~v and the thumb point in the direction of the vector ~u × ~v .
144 CHAPTER 5. INTRODUCTION TO GEOMETRY IN SPACE
If we denote by θ the angle between the vectors ~u and ~v and we use the formula
k~u × ~v k
sin(θ) = ·
k~ukk~v k
In this case we deduce that
k~u × ~v k = k~uk k~v k | sin(θ)|
Proof. 1. We consider the parallelogram that has ~u and ~v as adjacent sides. From
the previous chapter we knew that its area is defined by
Area of Parallelogram = k~uk k~v k | sin(θ) | = k~u × ~v k.
2. The proof of 2 is similar to 1.
As a second application, we use the cross product to compute the distance
between a fixed point to a line.
Let us consider a point A in space and a line (D). We agree that if the point A
belongs to the line (D), then the distance between A and (D) is equal to zero. Now
we assume that the line (D)
does not contain the point A and we aim to compute
the distance dist A , (D) .
To estimate this distance we draw the line (D0 ) perpendicular to (D) and passing
through the point A. We denote by B the intersection point of (D) and (D0 ). See
figure below
A
(D)
The distance between the point A and the line (D) is equal to the distance AB.
Now we take a point C in (D) which is different to B. The triangle ABC is a
−−→ −→
right triangle with hypothenus the segment line [AC]. We set θ := CB , CA . In
this case we have AB = AC · sin(θ). This means that
−→ −−→ −→ −−→
−−→ −→ AC BC sin(θ) AC × BC
AB = AC sin(θ) = −−→ = −−→ ·
BC BC
This proves the following lemma.
5.5. STUDY OF PLANES IN SPACE 145
Lemma 5.4.14. If (D) is the line that passes through the point B with direction
vector ~u, then, the distance between the point A to the line (D) is
−−→ →
AB × −
u
dist A , D = →
− · (5.4.4)
u
Example 5.4.15. To determine the distance between the point A(0, 1, −2) to the
line
x=1+t
(d) y = −3 − t t∈R
z = 2t
~i ~j ~k
−−→ −4 2~ 1 2 ~ 1 −4 ~
AB × ~u = 1 −4 2 = i− j+ k = −6~i + 3~k.
−1 2 1 2 1 −1
1 −1 2
Therefore we obtain
−−→ √ r r
AB × ~u 36 + 9 45 15
dist A, D = =√ = = ·
~u 1+1+4 6 2
Exercise 5.4.16. Determine the distance between the point A and the line (D)
x = −t
1. A (1, 0, −1) and y = 2 + t, t ∈ R,
z = 1 + 2t
x = 1 + 3t
2. A (2, 1, −3) and y = 2 − t, t ∈ R.
z = −2 t
Lemma 5.5.2. Let ~u and ~v be two noncollinear vectors in the plane P. Then,
the vector ~u × ~v is normal to the plane P.
In this paragraph we establish that one can define the equation of a plane by,
considering a point which belongs to the plane and a normal vector to this plane.
Now we consider a plane P which contains the point A (x0 , y0 , z0 ). We suppose
that the vector ~n a, b, c is normal to P. We take another point M (x, y, z) in P
−−→
which we assume different to A. Therefore, AM ⊥ ~n. This involves
−−→
~n · AM = 0. (5.5.1)
a x + b y + c z + d = 0. (5.5.2)
Example 5.5.4. To determine a cartesian of the plane P which contains the points
A(0, 0, 1), B(2, 0, 0), and C(0, 3, 0) we define a normal vector to P. As we we know
−−→ −→
that the vectors AB(2, 0, −1) and AC(0, 3, −1) are two non collinear vectors in P,
−−→ −→
the vector AB × AC (3, 2, 6) is normal to the plane P. Then, the cartesian equation
of P is
3x + 2y + 6z + d = 0.
Using the point A(0, 0, 1) we get
3x + 2y + 6z − 6 = 0.
5.5. STUDY OF PLANES IN SPACE 147
Definition 5.5.6. The system (5.5.3) is called parametric equations of the plane
P.
Theorem 5.5.7. If P is the plane with direction vectors ~u and ~v that contains
A(xA , yA , zA ), then parametric equations of P are defined by (5.5.3).
−−→ −−→
norm of the vector BC. This is nothing but the projection of the vector AB on the
vector ~n.
Since the triangle ABC is right and 0 ≤ θ ≤ π,one has
−−→ −−→
−−→ −−→ AB · ~n · sin(θ) AB × ~n
BC = AB sin(θ) = = · (5.5.4)
k~nk k~nk
Theorem 5.5.10. If P is the plane that is normal to ~n and contains A, then,
the distance between point B and P is defined by (5.5.4).
Example 5.5.11. To determine the distance between B(1, 0, 3) to the plane P :
3x + 2y + 6z − 6 = 0 , we have to identify a normal vector to P. From the cartesian
equation of P, we deduce that the vector ~n (3, 2, 6) is a normal vector. On the other
hand, A (0, 0, 1) ∈ P. Simple calculations show that
−−→ −−→
AB (1, 0, 2) and AB × ~n (−4, 0, 2).
Therefore, the distance between B to the plane P is
−−→ √ √
AB × ~n 16 + 4 20
d(B, P) = =√ = ·
k~nk 9 + 4 + 36 7
Exercise 5.5.12. Compute the distance between the point B and the plane P
1. B (0, 3, 2) and 3 x + 2y + 2 = 0,
2. B (1, −1, 1) and 5 x + y + 2 z = 0,
3. B (0, 5, 0) and z + 2 = 0.
Let P be a plane which is normal to the vector n (a, b, c). The plane P has
equation ax + by + cz + d = 0, with some real number d. We consider the point
B (x0 , y0 , z0 ) and we assume that B 6∈ P. In order to estimate the distance between
the point B to the plane P, we draw the line (d) orthogonal to P and passes through
the point B. We denote by C (xC , yC , zC ) the intersection point between (d) and
P.
−−→
The vectors ~n and CB are collinear. We suppose they have the same sense.
Therefore
−−→ −−→ −−→
CB · ~n = CB · ~n = CB ~n .
As we know that the distance between the plane P and the point B is equal to
−−→
CB , one deduces from the identity above that
−−→
CB · ~n
dist(B, P) = · (5.5.5)
k~nk
−−→
On the other hand we know that CB (x0 − xC , y0 − yC , z0 − zC ). Substituting
in (5.5.5), we obtain
−−→
CB · ~n = ax0 + by0 + cz0 − (axC + byC + czC ) (5.5.6)
Since C ∈ P, we have d = −(axC + byC + czC . Gathering (5.5.5) and (5.5.6) we
obtain
ax0 + by0 + cz0 + d
dist(B, P) = · (5.5.7)
k~nk
This leads to the following theorem
5.5. STUDY OF PLANES IN SPACE 149
Theorem 5.5.13. The distance between the point B and the plane P which has
cartesian equation ax + by + cz + d = 0 is given by (5.5.7).
Example 5.5.14. To find the distance between the point B (−1, 1, 0) and the plane
P : x + z + 2 = 0 we proceed as follows: we define the normal vector ~n (1 , 0 , 1)
and we apply (5.5.7). Thus we obtain
√
−1+0+2 2
dist B , P = √ =
1+0+1 2
Exercise 5.5.15. Estimate the distance between the point B to the plane P
1. B (0, 0, 3) and x − 2y + 2z − 1 = 0,
2. B (2, 1, 0) and − x + 2y + 2 z + 1 = 0.
We know that in the plane two lines can coincide, intersect. They can also be
parallel. Here we add that two lines are co-planar if they live in the same plane.
Now we consider two lines which are not co-planar. We assume they are not
parallel and they does not intersect. In this case we say they are skew lines.
Definition 5.5.16. Two lines are called skew lines if they satify these three
properties
We analyze their parametric equations. Hence, we define the vectors ~v1 (2, 4, 3)
and ~v2 (1, 3, 2) as respective direction vectors of (L1) and (L2).
We can observe that the vectors ~v1 and ~v2 are not collinear. Thus the lines (L1 )
and (L2 ) are not parallel.
Now we assume that (L1 ) and (L2 ) intersect. This means that there exist a point
having coordinates which satisfy the following system
4 + 2t = 2 + s,
−5 + 4t = −1 + 3s,
1 + 3t = 2s
150 CHAPTER 5. INTRODUCTION TO GEOMETRY IN SPACE
~ = ~u × ~v · →
−
~u, ~v , w w (5.5.8)
The identity (5.5.8) is called the mixed product of vectors ~u, ~v and w.
~ In the next
section we introduce and study the mixed product.
Definition 5.6.1. The mixed product of three vectors ~u, ~v and w ~ is defined by
~ = ~u × ~v · w
~u, ~v , w ~ = k~u × ~v k kwk
~ cos(~u × ~v , w).
~ (5.6.1)
u1 u2 u3
~ = v1
~u, ~v , w v2 v3 . (5.6.2)
w1 w2 w3
Using the fact that the vector ~u × ~v is orthogonal both to ~u and ~v , we obtain
~u, ~v , w
~ =w~ · ~u × ~v = 0.
5.6. SOME PROPERTIES OF THE MIXED PRODUCT 151
Conversely, if ~u × ~v · →
− →
−
deduces that w ⊥ u × ~v . Since on the other
w = 0, one
hand ~u ⊥ ~u × ~v and ~v ⊥ ~u × ~v , we conclude that there exist two real numbers
α and β such that
→
−
w = α~v + β~v .
This means that the vector ~u, ~v and → −
w are coplanar. This proves the following
theorem
Theorem 5.6.4. The vector ~u, ~v and w
~ are coplanar if and only if
~ · (~u × ~v ) = 0.
w (5.6.3)
We remind that the mixed product can be used to compute volumes of some
geometrical objects.
Indeed let ABCDEFGH be a solide with basis the parallelogram ABCD. The
volume of the solide ABCDEFGH is given by the following formula
Volume(ABGDEF GH) = Area(ABCD) × height.
−−→ −−→
In this particular case we have area of ABCD is AB × AD and the height is
−→ −→ −−→
given by AE · | cos(θ)|, where theta is the angle between the vectors EA and EK.
Hence, the volume is given by
−−→ −−→ −−→ −−→ −→
Volume(ABGDEF GH) = AB × AD AE · | cos(θ)| = AB × AD · AE .
This leads to the following theorem
Theorem 5.6.5. The volume of the parallelepiped with adjacent sides →
−
u, →−
v,
→
−
and w is
Volume = → −
u ×→ −
v ·→ −
w (5.6.4)
Example 5.6.7. To find the volume of the parallelpiped with adjacent sides
~a (1, 0, 2), ~b (−1, 1, 0) and ~c (2, 0, 0). We construct the matrix
1 0 2
A = −1 1 0 .
2 0 0
−1 1
We have det(A) = 2 = 2 · (−2) = −4 Hence, using (5.6.5) we have
2 0
Volume = det(A) = 4.
152 CHAPTER 5. INTRODUCTION TO GEOMETRY IN SPACE
then,
u1 v1 + u2 v2 + u3 v3 = 0.
Solution. We consider the vectors ~u u1 , u2 , u3 and ~v v1 , v2 , v3 , then,
2
= u21 + v12 + u22 + v22 + u23 + v32 + 2 u1 v1 + u2 v2 + u3 v3 ,
~u + ~v
2
= u21 + v12 + u22 + v22 + u23 + v32 − 2 u1 v1 + u2 v2 + u3 v3 .
~u − ~v
Since ~u + ~v = ~u − ~v we obtain
2 2
~u + ~v = ~u − ~v .
Consequently we have
u1 v1 + u2 v2 + u3 v3 = 0.
Exercise 5.7.2. Let ~a, ~b and ~c be three vector, such that,
~a = 3, ~b = 1 and ~c = 4.
~a + ~b + ~c = ~0.
Prove that
~a · ~b + ~b · ~c + ~c · ~a = −13.
Solution. We consider the vectors ~a, ~b and ~c, such that ~a = 3, ~b = 1 , ~c = 4
and
~a + ~b + ~c = ~0.
Then, we have
2 2 2 2
~a + ~b + ~c + ~b + 2 ~a · ~b + ~b · ~c + ~c · ~a
= ~a + ~c
= 9 + 1 + 16 + 2 ~a · ~b + ~b · ~c + ~c · ~a = 0.
2 ~a · ~b + ~b · ~c + ~c · ~a + 26 = 0 =⇒ ~a · ~b + ~b · ~c + ~c · ~a = −13
5.7. EXERCISE SESSION 153
Exercise 5.7.3. In each case determine the vectorial equation and a set of para-
metric equations for the line (d) that is parallel to ~u and contains the point A
1. A (−1, 0, 2) and ~u (1, 5, 4)
2. A (−3, −1, 2) and ~u (2, 5, −1)
3. A (1, −2, 6) and ~u (3, 5, −11)
Solution. 1. We consider the point A (−1 , 0 , 2) and the vector ~u = ~i+5 ~j −4 ~k.
In this case the vectorial equation of the line passing through the point A with
direction vector ~u is :
−→
~r(t) = OA + t~u = −~i + 2 ~k + t ~i + 5 ~j − 4 ~k
= − 1 + t ~i + 5 t ~j + 2 − 4 t ~k
2. For the line (d) that passes through the point A (3 , −1 , 2) with direction
vector ~u = 2~i + 5 ~j − ~k, we have
−→
~r(t) = OA + t~u = 3~i − ~j + 2 ~k + t 2~i + 5 ~j − ~k
= 3 + 2 t ~i + − 1 + 5 t ~j + 2 − t ~k
for some t ∈ R.
A set of parametric equations of this line is defined by this system
x = 3 + 2k
y = −1 + 5 k , k∈R
z = 2 − k.
= 1 + 3 t ~i + − 2 + 5 t ~j + 6 − 11 t ~k
where t ∈ R.
A parametric equation of the line passing through A with direction ~u is
x = 1 + 3 `,
y = −2 + 5 ` , `∈R
z = 6 − 11 `.
154 CHAPTER 5. INTRODUCTION TO GEOMETRY IN SPACE
Determine the coordinates of the point on the line that is the closest to the origine.
Solution. We consider the point O (0 , 0 , 0).We can observe that the point
A (2 , 3 , 1) is a point of the line;
−→
Then OA (2 , 3 , 1). The vector ~u (−3 , 1 , 1) is a direction vector of this line.
On the other hand we know that the distance between O and the line (∆) is
defined by the formula
−→
OA × ~u
d= ·
~u
Straightforward computations show that
~i ~j ~k
−→
OA × ~u = 2 3 1 = 2~i − 5 ~j + 11 ~k.
−3 1 1
Hence, we conclude that
√ √ r
4 + 25 + 121 150 6
d= √ = √ =5·
9+1+1 11 11
Exercise 5.7.5. Determine the cross (vectorial) product of the following vectors
1. ~a (4, 9, 1) and ~b (−3, −2, 5)
~i ~j ~k
~
~a × b = 3 0 0 = −6 ~j.
0 0 2
~i ~j ~k
~a × ~b = 1 1 0 = −2 ~k.
1 −1 0
Solution. Let ~u (−2 , 4 , 2) and ~v (3 , 2 , −3) be two vectors. We know that the
vector ~u × ~v is orthogonal both to ~u and ~v . Hence,
~i ~j ~k
~u × ~v = −2 4 2 = −16 ~i + ~k
3 2 −3
is orthogonal to ~u and ~v . To obtain a unit vector that has the same direction as
~u × ~v we define
√
−16 ~i + ~k
~u × ~v 2 ~ ~
~e = = √ =− i+k .
~u × ~v 16 2 2
−x + 2y + 3z + d = 0
−1 + 4 + 0 + d = 0 ⇒ d = −3
−x + 2 y + 3 z − 3 = 0.
156 CHAPTER 5. INTRODUCTION TO GEOMETRY IN SPACE
5.8 Problems
Problem 5.8.1. Define the scalar product of the following vectors
1. →
−
u 3 , 2 , −5 and →−
v 10 , 1 , 2
2. →
−
u 2 , 1 , 5 and →
−
v 7 , −9 , −1
3. →
−
u 2 , −1 , 1 and →−
v 0 , −1 , 1
4. →
−
u 1 , 2 , 3 and →
−
v 0, 2, 1
→
− →
−
Problem 5.8.2. We consider the vectors → −a and b such that →
−
a = 3 and b =
5. Determine the value of the real number α such that
→− →
− − →
−
a +α b ⊥ →
a −α b .
• →
−
x ⊥→
−
u
• →
−
x ·→
−
v = −6
Problem 5.8.7. Determine the area of the parallelogram which has adjacent sides
the vectors →
−
u and →
−
v
1. →
−
u = 2~i + 3~j + 5~k, and →
−
v = ~i + 2~j + ~k,
2. →
−
u = ~i + ~j, and →
−
v = −3~j + ~k.
5.8. PROBLEMS 157
P : x − 3y + 2z + 1 = 0.
a. Establish that the line d is orthogonal to the plane ABC .
b. Find a cartesian equation to the plane ABC .
3. Let H be the intersection point of the line d and the plane ABC .
a. Prove that
−−→ −−→ −−→ →−
2HA + HB − 2HC = 0 .
b. Determine the set of all points M such that
−−→ −−→ −−→ −−→ −−→
−2M A − M B + M C · M B − M C = 0.
2x + y − 2z + 4 = 0.
We consider the points A 3 , 2 , 6 , B 1 , 2 , 4 and C 4 , −2 , 5
1. Prove that the points A, B and C define a plane.
2. Show that this plane is identic to P.
3. Establish that the triangle ABC is right.
4. Define parametric equations of the line ∆ that passes through the point O and
perpendicular to P.
Problem 5.8.17. Let A 3 , 0 , 6 and I 0 , 0 , 6 be two points and D the line
that passes through A and I. We consider the planes
P : 2x + z − 6 = 0 and Q : y − 2z + 12 = 0.
5.8. PROBLEMS 159
1
x=t−
3
1 t ∈ R.
y=− ,
3
z=t
3. Determine the distance between the point A and the planes P and P 0 .
Contents
6.1 Cartesian Coordinate Systems in the plane . . . . . . . 162
6.1.1 Change of the Origin of a System . . . . . . . . . . . . . 163
6.1.2 Change of the Basis of a System . . . . . . . . . . . . . 164
6.2 Polar Coordinate System . . . . . . . . . . . . . . . . . . 165
6.2.1 Conversion From Polar to Cartesian Coordinates . . . . 165
6.2.2 Conversion From Cartesian to Polar . . . . . . . . . . . 168
6.3 Cylindrical Coordinate System . . . . . . . . . . . . . . 169
6.3.1 From Cylindrical to Cartesian coordinate System . . . . 170
6.3.2 From Cartesian to Cylindrical Coordinate System . . . 171
6.4 Spherical Coordinate System . . . . . . . . . . . . . . . 171
6.4.1 From Spherical To Cartesian Coordinates . . . . . . . . 172
6.4.2 From Cartesian To Spherical Coordinates . . . . . . . . 174
6.4.3 From Spherical To Cylindrical Coordinates . . . . . . . 175
6.4.4 From Cylindrical To Spherical Coordinates . . . . . . . 175
6.5 Circles and Spheres . . . . . . . . . . . . . . . . . . . . . 176
6.5.1 Equations of Circles . . . . . . . . . . . . . . . . . . . . 176
6.5.2 Equation of Spheres . . . . . . . . . . . . . . . . . . . . 177
6.6 Problems . . . . . . . . . . . . . . . . . . . . . . . . . . . 179
161
162 CHAPTER 6. INTRODUCTION TO COORDINATE SYSTEMS
of the coordinates. To explain that quickly, we consider a point A in the plane which
is turning around a fixed point B.To follow the trajectory of the movement of the
point A it is better to locate the point A by the angle θ, (θ is the angle between the
−−→ −−→
vector BA and the x-axis) and r = BA . In this case we represent the coordinates
of the point A by the ordered pair (r, θ). This representation is called the polar
coordinates of A.
In this chapter we will define some coordinate systems and study the relations
that can exist between them.
In the first part we study different coordinate systems in the plane. In part 2
we introduce coordinate systems in the space. In the last part we investigate circles
and the spheres.
y − axis
O x − axis
Now we take a point A in the plane. To determine the coordinates of the pont
A we draw a vertical line that pass through A. We denote by xA the intersection
of this vertical line and the x-axis. We draw again a horizontal line which passes
through the point A and we denote yA its intersection with the y-axis. The ordered
couple (xA , yA ) is called the coordinates of the point A in the cartesian system
(O, ~e1 , ~e2 ). We specify that the expression A (xA , yA ) is equivalent to
xA
(xA , yA ) = = xA~e1 + yA~e2 .
yA
6.1. CARTESIAN COORDINATE SYSTEMS IN THE PLANE 163
y − axis
yA +A (xA , yA )
~e2
O ~e1 xA x − axis
Throughout this chapter frame and coordinate system have the same meaning. Let
P be a point in the plane with coordinates (xP , yP ) in the frame (O, ~e1 , ~e2 ). We
remark that if we change either the origin of the frame or the basis of the frame,
the coordinates of P should change. In the next subsection we investigate how the
change of origin can affect the coordinates of the points and then vectors.
~e2
yA A+
~e1
~e2
O ~e1 xA
Theorem 6.1.2. Let (O, ~e1 , ~e2 ) and (A, ~e1 , ~e2 ) be two cartesian systems in the
plane, such that, A (xA , yA ) in (O, ~e1 , ~e2 ). We assume that M has coordinates
(x, y) in (O, ~e1 , ~e2 ). Then, the coordinates of M in (A, ~e1 , ~e2 ) are given by (6.1.1).
Instead of changing the origin, we can decide to keep the same origin for the
two systems and change the bases in this way : let M be a point in the plane, we
want to find the coordinates of M in the system (O, ~e1 , ~e2 ) as well as in the system
O, f~1 , f~2 , where f~1 , f~2 is another basis in the plane. In the next subsection we
the ordered pairs (~e1 , ~e2 ) and f~1 , f~2 stand for bases in the plane. There are four
real numbers a, b, c and d, such that, α = ad − bc 6= 0 and f~1 = a ~e1 + b ~e2 and
f~2 = c ~e1 + d ~e2 . If M is a point in the plane with coordinates(x, y) in the system
(O, ~e1 , ~e2 ) and (X, Y ) in the system (O, f~1 , f~2 ), then we obtain
−−→
OM = x ~e1 + y ~e2 = X f~1 + Y f~2 .
Our main purpose is to determine the expressions of X and Y . So replacing f~1
−−→
and f~2 by their expressions,on has OM = X (a ~e1 + b ~e2 ) + Y (c ~e1 + d ~e2 ). This leads
to the identity x~e1 + y~e2 = (a X + c Y ) ~e1 + (b X + d Y ) ~e2 . This provides
x = aX + cY
(6.1.2)
y = b X + d Y.
Multiplying by d the first equation of the system (6.1.2) and by c the second equa-
tion, one gets dx − cy = αX, where we recall that α = ad − bc.
Now, we multiply the first equation of the system (6.1.2) by b and the second
equation by a. This involves, −bx + ay = αY. Therefore, we obtain
X = α−1 d x − c y
(6.1.3)
Y = α−1 − b x + a y .
Theorem 6.1.3. Let (O, ~e1 , ~e2 ) and O, f~1 , f~2 be two coordinate systems, such
that, f~1 = a~e1 + b~e2 and f~2 = c~e1 + d~e2 with four real numbers a, b, c and d
which satisfy α = ad − bc 6= 0. We assume that M has coordinates (x, y) in the
system (O, ~e1 , ~e2 ). Then, the coordinates of M in the system O, f~1 , f~2 are given
by (6.1.3).
A
×
θ
O
θ
O polar-axis
yA A
×
~e2 θ
O ~e1 xA
166 CHAPTER 6. INTRODUCTION TO COORDINATE SYSTEMS
The polar coordinates of A are defined by A (r, θ). Now we draw the vertical
line that passes through the the point A. The intersection of that line with the
x-axis is the point (xA , 0). On the other hand (0, yA ) is the intersection point of the
horizontal line which passes through A and the y-axis. We observe that the triangle
which has vertices the point (xA , yA ), (xA , 0) and (0 , 0) is right with hypothenus
the segment line [OA]. This involves xA = r cos(θ) and yA = r sin(θ). Hence, this
holds the following lemma:
Lemma 6.2.2. The cartesian coordinates of the point A which has polar coordi-
nates (r, θ) are
x = r cos(θ)
(6.2.1)
y = r sin(θ).
The functions cos and sin are 2π-periodic functions. Consequently, (r, θ) and (r, θ +
2π) represent the same point.
Lemma 6.2.5. For every integer k the couples (r, θ) and (r, θ + 2kπ) represent
the same point in the plane.
Remark 6.2.6. The most important fact that emphasize the lemma above, is the
non-uniqueness of polar coordinates for any given point.
6.2. POLAR COORDINATE SYSTEM 167
From now on we mean by the expression A (r, θ), the point A which has polar
coordinates (r, θ).
√ √
Example 6.2.7. The following polar coordinates 3, −π/3 , 3, 5π/3 , and
√
3, 11π/3 represent the same point.
Lemma 6.2.8. The point A (r, θ) and A0 (r, −θ) are symetric with respect to the
x-axis
A
×
θ
O
−θ
r
A0
×
Proof. Take the point A (r, θ). The associated cartesian coordinates of this point
are (r cos(θ), r sin(θ)). On the other hand we know that the cartesian coordinates
of the point A0 (r, −θ) are (r cos(θ), −r sin(θ)). As we can observe it
Lemma 6.2.9. The points A (r, θ) and A0 (r, π − θ) are symetric with respect to
the y-axis.
Proof. Let A (r, θ) and A0 (r, π − θ) be two points in the plane. Then, the points A
and A0 have respectively cartesian coordinates
This illustrates that xA0 = −xA and yA0 = yA . Hence the points A and A0 are
symetric with respect to the y-axis.
Lemma 6.2.10. We consider the points A and A0 with respective polar coordi-
nates (r, θ) and (r, π + θ). Then A and A0 are symetric with respect to the origin
O.
Proof. Take A (r, θ) and A0 (r, π +θ). Therefore we have xA0 = −xA and yA0 = −yA .
Because cos(π + θ) = − cos(θ) and sin(π + θ) = − sin(θ). This proves that A and
A0 are symetric with respect to the origin O.
When, two angles θ and θ0 satisfy the identity θ + θ0 = π/2, then, one has
Definition 6.2.11. We call first bisector the line that has cartesian equation
y = x.
It is well known that Two points A (x, y) and A0 (x0 , y 0 ) are symetric with respect
to the first bisector if and only if
x0 = y and y 0 = x.
Theorem 6.2.12. If θ ∈ [0, π/2], then, the points A (r, θ) and A0 (r, π/2 − θ) are
symetric with respect to the first bisector.
Proof. We consider the points A (r, θ) and A0 (r, π/2 − θ). The cartesian coordinates
associated respectively to A and A0 are
hand, we have cos(θ) = xA /r and sin(θ) = yA /r. To find the value of θ one can use
as he pleased one of the following functions arctan, arccos or arcsin .
Lemma 6.2.13. For any point A (xA , yA ) in the plane, the polar coordinates of
A are p
r = x2 + y 2 ,
θ,
• h is a real number.
170 CHAPTER 6. INTRODUCTION TO COORDINATE SYSTEMS
Notation: The point A which has cylindrical coordinates ρ , θ , h is denoted
A ρ, θ, h .
√
Example 6.3.2. For instance we consider the point A 2, π3 , −1 and B 3, π4 , 2).
have
−−→
OM = x~i + y~j + z~k (6.3.2)
Identifying (6.3.1) and (6.3.2) we deduce the relations between cartesian coordinates
of M and the cylindrical coordinates
Lemma 6.3.3. Let A (ρ, θ, h) be a point. Then, the cartesian coordinates of A
are
x = ρ cos(θ)
y = ρ sin(θ) (6.3.3)
z = h.
Given a point A with cartesian coordinates (x, y, z), One may wonder, if we can
determine the cylindrical coordinates of the point A.
6.4. SPHERICAL COORDINATE SYSTEM 171
Step 1: we define h := z.
1. A (−1, 1, 3),
√
2. B (1, − 3, 6),
In the three dimensional space we can define another coordinate system called
the spherical coordinate system.
We define
−−→
• r = OM the distance between the points O and M
−−→ −−→
• θ = ~uz , OM , as the angle between ~uz and the vectors OM . This angle is
always supposed to belong to the interval [0, π].
−−→ −−→
• ϕ = ~ux , OP , is the angle between the vectors ~ux and OP . The angle ϕ is
in [0, 2π].
To catch the real meaning of the name spherical coordinates, we figure out that the
point M is moving on the sphere of center O and radious r. This is illustrated
in
the figure above. The point M can be located by the ordered triple r , θ , ϕ .
Definition 6.4.1. The spherical coordinates of the point M are r, θ, ϕ .
OP = OH = r sin(θ). (6.4.1)
6.4. SPHERICAL COORDINATE SYSTEM 173
See figure above. Since P is the orthogonal prejection of M in the plane xOy, then,
M and P share these properties x = xp = xM and y = yP = yM .
On the other hand we know that
x = OP cos(ϕ)
y = OP sin(ϕ).
ply the lemma above. One deduces that the cartesian coordinates of A are defined
by √ √
x = 3 sin π
6 · cos π
4 = 3 · 21 · 22 = 3 4 2
√ √
y = 3 sin π6 · sin π4 = 3 12 · 22 = 3 4 2
√ √
z = 3 cos π6 = 3 23 = 3 2 3 ·
√ π 5π
Example 6.4.5. To determine the cartesian coordinates of the point B 2, 3 , 6 ·
we write
√
π
√ √ √
5π 3 − 3
√
−3 2
x = 2 sin 3 · cos 6 = 2 · 2 · 2 = 4
√ √ √ √
y = 2 sin π3 · sin 5π 6 = 2 23 · 21 = 46
√ √ √
z = 2 cos π = 2 1 = 2 ·
3 2 2
5. E (1, 0, 0).
One may ask if the converse operation is possible. In other words is it possible
to define the spherical coordinate of a point or a vector from the cartesian one.
174 CHAPTER 6. INTRODUCTION TO COORDINATE SYSTEMS
ϕ such that,
y
cos(ϕ) = √ 2x sin(ϕ) = √ ·
and
x +y 2 x2 +y 2
√
Example 6.4.8. We consider the point A (1, −1, 2). Then, we have
√
√ 2
r= 1+1+2=2 and θ = arccos = π/4.
2
Because the angle θ belongs to [0, π].
On the other hand, we have
√ √
√ 2 − 2
ρ= 2, cos(ϕ) = and sin(ϕ) = ·
2 2
Hence, we obtain ϕ = − π4 ·
We conclude that the spherical coordinates of A are 2, π4 , −π
4 ·
Therefore ρ = r| sin(θ)| = r sin(θ), because θ ∈ [0, π]. This leads to the following
theorem
Theorem 6.4.10. The cylindrical coordinates of the point A (r, θ, ϕ) are defined
by
r sin(θ), ϕ, r cos(θ) (6.4.4)
Theorem 6.5.4. The polar parametric equations of the the circle C (I; R) with
center Let I (x0 , y0 ) and radius R > 0 are
x − x0 = R cos(θ)
y − y0 = R sin(θ),
Proof. Consider the point M (x, y) in the plane and the frame (I,~i, ~j). We define θ
−−→
as the angle between ~i and IM and IM = R. Then,we get
−−→
IM = R cos(θ)~i + R sin(θ) ~j.
−−→
Since IM (x − x0 , y − y0 ), we deduce from the uniqueness of coordinates that
x − x0 = R cos(θ)
y − y0 = R sin(θ),
Example 6.5.5. For instance take B (1, 2) and R = 2. Then, the polar parametric
equations of the circle C (B; 2) are
x − 1 = 2 cos(θ)
y − 2 = 2 sin(θ),
where θ ∈ R.
Exercise 6.5.6. Let A (1, 0), B (2, 3) and C (3, 1) be three points in the plane. Find
the polar parametric equations of the following circles
1. C (A; 3),
2. C (B; 6),
3. C (C; 10).
Theorem 6.5.7. The cartesian equation of the sphere S (A; R) with center
A (x0 , y0 , z0 ) radius R is
x2 + y 2 + z 2 − 2 (x · x0 + y · y0 + z · z0 ) + d = R,
x2 + y 2 + z 2 = 36.
Exercise 6.5.9. We consider the points A (1, 2, 0), B (3, 2, 0), C (0, 1, 5), D (3, 4, 1)
and E (−1, 0, 3). Give the cartesian equations of the spheres
1. S (A; 2),
2. S (B; 5),
3. S (C; 4),
4. S (D; 6),
5. S (E; 3).
Now we consider the points A (x0 , y0 , z0 ) and M (x, y, z). We define the vector
−−→
AM . Furthermore we assume, the point M belongs to S(A; R). Using cylindrical
coordinates system we can find R > 0, an angle θ and h ∈ R, such that R2 > h2
and √
2 2
x − x0 = R − h cos(θ)
√
y − y0 = R2 − h2 sin(θ) (6.5.2)
z − z0 = h.
The system (6.5.2) is called the parametric equations of the sphere in cylindrical
coordinate system.
Theorem 6.5.10. The parametric equations of the sphere S(A; R) with center
A (x0 , y0 , z0 ) and radius R > 0 in a cylindrical coordinate system are given by
(6.5.2).
Theorem 6.5.11. The parametric equations of the sphere S(A; R) with center
A (x0 , y0 , z0 ) and radius R in spherical coordinates are
x − x0 = R sin(θ) cos(ϕ)
y − y0 = R sin(θ) sin(ϕ)
z − z0 = R cos(θ),
Proof. We consider the frame (A,~i, ~j, ~k). Let (R, θ, φ) be the spherical coordinates
of M in this frame. Then we have
x − x0 = R sin(θ) cos(ϕ)
y − y0 = R sin(θ) sin(ϕ)
z − z0 = R cos(θ).
6.6 Problems
Problem 6.6.1. For each of the following points define its polar coordinates
1. 1 ; −1
√ √
2. 2; 2
√
3. 3 ; −1
√
4. 1 , 3
5. − 1 , −1
6. − 3 , 0
7. 0 , −5 .
Problem 6.6.7. Write the cartesian equation of the circle with center the point A
and radius R :
1. A (2 ; 3) and R = 3
√
2. A (−1 ; 3) and R = 2
√
3. A (0 ; 1) and R = 5
√
4. A (0 ; 0) and R = 7
5. A (−1 ; 0) and R = 5
Problem 6.6.8. Find the center and the radius of the following circles
1. x2 − 6x + y 2 − 8y = 27
2. x2 + y 2 + 2y = 7
3. x2 + y 2 + 2x = 2
4. x2 + y 2 − 2y = 3
5. x2 + y 2 + 3x = 3.
Problem 6.6.9. Define the cartesian equation of the sphere with center the point
A and radius R :
1. A (2 ; −1 ; 3) and R = 4
√
2. A (−1 ; 3 ; 0) and R = 3
√
3. A (0 ; 1 ; −2) and R = 17
√
4. A (0 ; 0 ; 1) and R = 6
5. A (−1 ; 0 ; 0) and R = 3
6. A (1 ; 1 ; 1) and R = 7
182 CHAPTER 6. INTRODUCTION TO COORDINATE SYSTEMS
7. A (2 ; −1 ; 3) and R = 5
Problem 6.6.10. Determine the center and the radius of the following spheres
1. x2 + y 2 + z 2 + 2z = 5
2. x2 + y 2 + z 2 − 4y = 5
3. x2 + y 2 + z 2 − 6x − 2y + 2z = 13
4. x2 + y 2 + z 2 = 3
5. x2 + y 2 + z 2 + 23 z = 1.
APPENDIX A
SECOND ORDER POLYNOMIAL
FUNCTIONS
The main objective of this chapter is to remind some properties of second order
polynomial functions. We will define the canonical form of a second order polyno-
mial function. Variations and signs of such function will be studied.
f (x) = a x2 + b x + c.
In the following section we establish that every second order polynomial can be
written in the form
f (x) = AX 2 + B.
b b b2 b2 c
f (x) = ax2 + 2 x + c = a x2 + 2 x + 2 − 2 +
2 2a 4a 4a a
2 b b2 b2 b 2 b2 − 4ac
=a x +2 x+ 2 − +c=a x− − −
2a 4a 4a 2a 4a
b ∆
Setting α = − , ∆ = b2 − 4ac and β = − , we obtain f (x) = a(x − α)2 + β.
2a 4a
183
184 APPENDIX A. SECOND ORDER POLYNOMIAL FUNCTIONS
Definition A.1.1. Let f (x) = ax2 + bx + c be such that a 6= 0. The canonical form
of f is defined by
f (x) = a(x − α)2 + β
where we set
b b2 − 4ac
α := − and β=− ·
2a 4a
We can remark that, β = f (α).
Example A.1.2. We consider f (x) = 3x2 + 5x + 2. We have
5 25 − 4 × 3 × 2 −1
α=− and β = − = ·
6 4×3 12
2
5 1
Therefore f (x) = 3 x + − ·
6 12
Example A.1.3. Let f (x) = x2 − 2x + 1. In this case we have
−2 4−4×1×1
α=− =1 and β=− = 0·
2 4×1
Then f (x) = (x − 1)2 + 0 = (x − 1)2 .
Exercise A.1.4. Define the canonical form of the following functions
1. f (x) = x2 + 5x + 1
2. g(x) = x2 + 3
3. i(x) = 4x2 + 9x
4. j(x) = x2 − 6x + 9.
Let f (x) = ax2 + bx + c be such that, a 6= 0. There exist two real numbers α
and β, , such that, f (x) = a(x − α)2 + β.This, means that the identity f (x) = 0 is
equivalent to a(x − α)2 + β = 0. That is,
β
(x − α)2 = − ·
a
Considering the definitions of α and β, one obtains
b2 − 4ac
(x − α)2 = ·
4a2
The denominator of the right hand side part 4a2 > 0. Then, according to the sign
of b2 − 4ac, three cases hold:
Case 1: b2 − 4ac > 0. Then, we have two real roots:
√ √
∆ ∆
x1 = α − or x2 = α + ·
2a 2a
ax2 + bx + c = 0 (A.2.1)
∆ = b2 − 4ac.
∆ = 16 − 12 = 4 > 0.
1. x2 + 5x + 3 = 0
2. 2x2 + 5x + 2 = 0
3. x2 − 4x + 4 = 0
4. x2 + 2x + 3 = 0
5. x4 + 6x2 + 8 = 0.
Let f (x) = ax2 + bx + c be
√
such that a 6= 0√and ∆ = b2 − 4ac ≥ 0. Then there
exist real solutions x1 = 2a and x2 = −b+
−b− ∆
2a
∆
· We set
b c
S = x1 + x2 = − and P = x1 · x2 = (A.2.2)
a a
Theorem A.2.2. Let f (x) = ax2 + bx + c be such that a 6= 0 and ∆ = b2 − 4ac ≥ 0.
We consider S and P defined by (A.2.2). Then,
f (x) = a x2 − Sx + P .
From this, we deduce that, finding the solutions of the equation ax2 +bx+c = 0,
is equivalent to consider the following system
b
x1 + x2 = − a
x1 · x2 = c ·
a
Lemma A.2.3. Let f (x) = ax2 + bx + c be such that a 6= 0 and ∆ = b2 − 4ac ≥ 0.
Then, if x1 and x2 denote the real roots of f we have
In the next section we show how to study the sign of a second order polynomial
function.
Let f be an affine function. Using the definition above we can see that f (x) = 0
implies x = − ab · According to the sign of the real number a we deduce this sign
table
x −∞ −b/a +∞
ax + b −sign of a 0 sign of a
x −∞ −1/2 +∞
2x + 1 − 0 +
x −∞ 7/3 +∞
−3x + 7 + 0 −
1. f (x) = −x + 2
2. g(x) = 7x
3. h(x) = 6x + 13
4. j(x) = −5x.
∆ = b2 − 4 · a · b. (A.3.2)
Case 1: ∆ > 0. When ∆ > 0, we obtain two real roots x1 and x2 . Here we assume
x1 < x2 . Then we have this sign table
x −∞ x1 x2 +∞
2
x + 6x + 5 sign(a) 0 −sign(a) 0 sign(a)
x −∞ −5 −1 +∞
2
x + 6x + 5 + 0 − 0 +
188 APPENDIX A. SECOND ORDER POLYNOMIAL FUNCTIONS
b
Case 2: ∆ = 0. When ∆ = 0, we have only one real root x0 = − 2a · In this case
the sign table of f is given by
x −∞ −b/2a +∞
ax2 + bx + c sign(a) 0 sign(a)
x −∞ 1 +∞
−x2 + 2x − 1 − 0 −
Case 3: ∆ < 0. In this case there are no real roots and the sign table is the fol-
lowing one
x −∞ +∞
ax2 + bx + c sign(a)
x −∞ +∞
2
x + 2x + 6 +
Theorem A.4.4. Let f (x) = ax2 + bx + c be such that, a < 0. Then f is increasing
b
in ]−∞ ; α] and decreasing in [α ; +∞[, where α = − 2a · Thus we have the following
variational table
x −∞ α +∞
β
f (x) % &
−∞ −∞
Since f is increasing in ] − ∞ ; α] and decreasing in [α ; +∞[, one deduce that,
f has a maximum at α which is equal to f (α) = β. we deduce this lemma.
Theorem A.4.5. Let f (x) = ax2 + bx + c be such that a < 0. Then f has a
maximum at α. The value of this maximum is f (α) = β.
Example A.4.6. Let f (x) = −2x2 + x + 1. The domain of f is Df = R. We have
b
α = − 2a = 14 and β = − 4a∆
= 98 · Then,
2
1 9
f (x) = −2 x − + ·
4 8
The function f is increasing in −∞ ; 4 and f is decreasing in 14 ; +∞ . Therefore
1
Here we give the three exams given in the academic year 2021-2022 and there
corrections.
Midterm
Exercise 1. Find the solutions of the following equations
(1). 2z 2 − 2z + 5 = 0, (2). z 2 + 2z − 5 = 0, (3). z 2 + z − 1 = 0,
(4). 3z 2 − 5z + 3 = 0, (5). z 4 + 5z 2 + 4 = 0, (6). z 4 − 3z 2 − 4 = 0.
√ √ √
Exercise 2. Let z = 1 + i 3 and z 0 = 1 − 3 + 1 + 3 i be two complex numbers.
1. Find the modulus and the argument of z.
2. Determine the polar form of z.
3. Deduce from question 2 the polar form of z 4 .
−z
4. Now we define the complex number z1 = 0 · Give the algebraic form of
z −z
z1 .
√
1+i 3
Exercise 3. We consider the complex number z = 1−i ·
191
192 APPENDIX B. EXAMS
∆ = 4 − 4 × 2 × 5 = −36 < 0.
Z 2 + 5Z + 4 = 0.
z1 = −2i, z2 = 2i, z3 = −i or z4 = i.
t2 − 3t − 4 = 0.
z1 = −i, z2 = i, z3 = −2 and z4 = 2.
193
√
Exercise 2. We consider
√ the complex
√ number z = 1 + i 3.
1. We have |z| = 12 + 3 = 4 = 2. Since
√
1 3
cos(θ) = and sin(θ) = ,
2 2
we deduce that θ = π3 ·
2. In this case the polar form of z is defined by
π π
z = 2 cos + i sin .
3 3
3. Using De Moivre’s Formula, we obtain
π π 4
4 4
4π 4π
z = 2 cos + i sin = 16 cos + i sin .
3 3 3 3
4. We define the complex number
√ √
−1 − i 3 −1 − i 3
z1 = √ √ √ = √
1− 3+i 1+ 3 −1−i 3 − 3+i
√ √ √ √
−1−i 3 − 3−i 3 + i + 3i − 3
= = =i
4 4
The complex number z1 is purely imaginary number.
√
1+i 3
√
Exercise 3. We define the complex number z = 1−i · Here we fix z1 = 1 + i 3
and z2 = 1 − i.
1. Then, we have
π
z1 = 2 and arg z1 =
3
√ π
z2 = 2 and arg z2 = − ·
4
From this we deduce that
√
z1 2 √
z = = = 2.
z2 2
Exercise 4. The aim of this exercise is to determine the solutions of the equation
z 4 + 6z 2 + 25 = 0.
t2 + 6t + 25 = 0.
Substitution Exam
Exercise 1. Find the solutions of the following equations
z 4 + 4 z 2 + 16 = 0. (B.0.3)
t2 − 4t + 3 = 0.
Exercise 3. We remind that the main objective of this exercise is to find the
solutions to this equation
z 4 + 4z 2 + 16 = 0.
To this end we make the following substitution Z = z 2 .
1. Now, we consider the equation Z 2 +4Z+16 = 0. We define ∆ = 16−64 = −48.
Therefore, we have
√ √
−4 − 4i 3 √ −4 + 4i 3 √
Z1 = = −2 − 2i 3 or Z2 = = −2 + 2i 3.
2 2
π
2.Since ω ∈ C such that |ω| = 2 and arg(ω) = 3,we can write
√ !
h π π i 1 3 √
ω = 2 cos + i sin =2 +i = 1 + i 3.
3 3 2 2
x4 + 4z 2 + 16 = 0 ⇐⇒ z 4 + 4z 2 + 16 = 0.
Final Exam
Exercise 1. In the three dimensional
space we consider the points A 3; 0; 0 ,
B 0; 6; 0 , C 0; 0; 4 and D − 5; 0; 1 .
4
1. Prove that the vector ~n 2 is normal to the plane ABC .
3
2. Determine a cartesian equation to the plane ABC .
3. Determine a set of parametric equations for the line ∆ that passes through
the point D and orthogonal to the plane ABC
4. Find the distance between the plane ABC and the point D.
Exercise 2. In the three
dimensional space we consider the points A 5; −5; 2 ,
B − 1; 1; 0 , C 0; 1; 2 and D 6; 6; −1 .
1. Determine the area of the triangle BCD
−2
2. Prove that the vector ~n 3 is normal to the plane BCD .
1
3. Determine a cartesian equation to the plane BCD .
4. Define a set of parametric equations to the line AB .
5. Find the volume of the tetrahedron ABCD.
Exercise 3. For any natural number n ≥ 1. we consider the sequence
1 3 5 2n − 1
vn = · · · ··· · ·
2 4 6 2n
1. Using the method of mathematical induction prove that for every n ≥ 1, we
have
1
0 ≤ vn ≤ √ ·
2n + 1
2. Compute limn→+∞ vn .
Exercise 4. We consider the sequence of complex numbers (zn ) defined by z0 = 1
and for any n ≥ 0,
√ !
3 3
zn+1 = + i zn .
4 4
We define the real sequence rn by rn = zn for every n ≥ 0.
√
3 3
1. Determine the modulus of the complex number 4 + 4 i·
√
3
2. Prove that the sequence (rn ) is geometric with common ratio and specify
2
r0
3. Determine the expression of the term rn for any n ≥ 0 and the
lim rn .
n→∞
200 APPENDIX B. EXAMS
24 · (−5) + 12 · 0 + 18 · 1 − 72 √
dist ABC , D = √ = 29·
2 2
24 + 12 + 18 2
Exercise 2. We consider four points A 5 ; −5 ; 2 , B − 1 ; 1 ; 0 , C 0 ; 1 ; 2
−−→ −−→
and D 6 ; 6 ; −1 . Then, we have BC (1 ; 0 ; 2) and BD (7 ; 5 ; −1).
201
−−→ −−→
1. Using the coordinates of the vectors BC and BD we obtain
~i ~j ~k
−−→ −−→
BC × BD = 1 0 2 = −10~i + 15~j + 5~k.
7 5 −1
From this, we get that the area of the triangle BCD is:
√
1 −−→ −−→ 1p 2 2 2
5 14
ABCD = BC × BD = (−10) + 15 + 5 = ·
2 2 2
−−→ −−→
2. Since BC and BD are not collinear they define the plane BCD . Any vector
of this plane should be a combination of these two vectors. To show that ~n is nornal
to the plane BCD we have to establish that
−−→ −−→
~n ⊥ BC and ~n ⊥ BD.
−2x + 3y + z + d = 0.
−2 · (−1) + 3 · 1 + 1 · 0 + d = 0 =⇒ d = −5.
−2x + 3y + z − 5 = 0.
−−→
4.The vector AB (−6 ; 6 ; −2) is a direction vector of the line AB . Using the
point A (5 ; −5 ; 2), we can see that
x = 5 − 6t,
y = −5 + 6t, t ∈ R.
z = 2 − 2t
is a set of parametric equations of AB .
5.The volume of the tatrahedron ABCD is determined by the formula
1 −−→ −−→ −−→
VABCD = AB · BC × BD .
6
As we know that
−6 6 −2
−−→ −−→ −−→
AB · BC × BD = 1 0 2 = 60 + 90 − 10 = 140.
7 5 −1
202 APPENDIX B. EXAMS
This involves
1 70
VABCD = · |140| = ·
6 3
1 3 5 2n − 1
vn = · · · ··· · ·
2 4 6 2n
1
Step 1. for n = 1, we have v1 = 2 > 0. Since
√ 1 1
3 ≤ 2 =⇒ v1 = <√ ·
2 3
Step 2. We suppose that the property is true for some natural number k ≥ 1. That
is,
1 3 2k − 1 1
0 ≤ vk = · · · · · · ≤√ ·
2 4 2k 2k + 1
Now, we have to prove that the property is true for k + 1. In other words, we have
to show that
1 3 2k − 1 2k + 1 1
0 ≤ vk+1 = · · ··· · · ≤√ ·
2 4 2k 2k + 2 2k + 3
We know that,
√
1 3 2k − 1 2k + 1 1 2k + 1 2k + 1
vk+1 = · · ··· · · ≤√ · = ·
2 4 2k 2k + 2 2k + 1 2k + 2 2k +2
√
2k+1 √ 1
Now we have to compare the positive real numbers 2k+2 and 2k+3
· We
have
√ √
1 2k + 1 2k + 1
√ =p =√
2k + 3 (2k + 3)(2k + 1) 2
4k + 8k + 3
√ √
2k + 1 2k + 1
and =√ ·
2k + 2 2
4k + 8k + 4
1
lim 0 = lim √ = 0.
n→+∞ n→+∞ 2n + 1
lim vn = 0.
n→+∞
2.From the definition of the sequence zn n≥0
, we can see that
√ ! √
3 3 3 3
rn+1 = zn+1 = + i zn = + i · zn
4 4 4 4
√
3
rn ·
2
We interprate this identity by saying that the sequence rn n≥0
is geometric with
√
3
common ration q = 2 and initial term r0 = |z0 | = 1.
√
3
3. Since rn n≥0 is geometric with common ration q = 2 and initial term
r0 = |z0 | = 1, we can write
√ !n √ !n
3 3
rn = r0 · q n = 1 · = , ∀n ≥ 0.
2 2
√
3
As we know that 0 < 2 < 1, then,
√ !n
3
lim rn = lim = 0.
n→+∞ n→+∞ 2
204 APPENDIX B. EXAMS
BIBLIOGRAPHY
205